Comprehensive Review in Clinical Neurology: A Multiple Choice Question Book for the Wards and Boards

Chapter 10. Neuromuscular II (Adult and Pediatric Muscle, Autonomic Nervous System, and Neuromuscular Junction Disorders)

Question

Questions 1–3

1. A 25-year-old man presents frequently to the Emergency Department with episodes of weakness and inability to move his arms and legs. Extraocular movements and bulbar and respiratory muscles are spared. The patient says that he gets these episodes with emotional stressors, after exercise, or after eating heavy desserts. On further questioning, he says that his father has similar episodes. Which of the following is the most likely diagnosis?

a. Hyperkalemic periodic paralysis

b. Hypokalemic periodic paralysis

c. Andersen-Tawil syndrome

d. Paramyotonia congenita

e. Myotonia congenita

2. A 42-year-old patient with chronic asthma has been treated with oral prednisone for over a year, given difficulties in controlling her illness. She comes with proximal weakness, and is noticed to have a cushingoid appearance. Which of the following is incorrect regarding the most likely cause of this patient’s weakness?

a. There is atrophy of type II fibers

b. Steroid dose reduction is important in the treatment of this condition

c. Creatine kinase levels are usually normal

d. Patients with Cushing disease may have proximal weakness

e. EMG shows specific findings that suggest this condition

3. A floppy baby is brought for evaluation, and on the basis of the biopsy, he is diagnosed with a centronuclear myopathy. Which of the following characteristics are incorrect regarding this condition?

a. Respiratory failure occurs in severe forms

b. The muscle fibers occurs show central nucleation

c. Pharyngeal and laryngeal muscles may be affected

d. Extraocular movements are rarely affected

e. Could be autosomal dominant, autosomal recessive, or X-linked

Questions 4–6

4. A 32-year-old woman presents to the clinic complaining of double vision of 1-month duration. She also reports her speech slurs when she talks for prolonged periods of time, and her eyelids start to droop toward the end of the day. On examination, she has bilateral ptosis that worsens with sustained upward gaze, she is unable to hold air in her mouth against resistance, and her neck flexors are weak. Which of the following is incorrect regarding this patient’s disease?

a. It is more common in males, and females tend to present at an earlier age as compared with males

b. The majority of patients have symptoms restricted to their extraocular muscles for the duration of their illness

c. Patients with human leukocyte antigen DR2 and DR3 are at a higher risk of developing this disorder

d. Family members of patients with this disease are at an increased risk of having this disorder

e. Patients with this disorder are at an increased risk of developing other autoimmune disorders

5. Regarding the pathophysiology of myasthenia gravis, which of the following is correct?

a. In its most common form, it is autoimmune due to antibodies against presynaptic voltage-gated calcium channels

b. It is due to inhibition of exocytosis of presynaptic vesicles containing acetylcholine

c. In its most common form, it is autoimmune due to antibodies against the acetylcholine receptor

d. It results from increased degradation of acetylcholine at the neuromuscular junction

e. In its most common form, it is due to mutations in the acetylcholine receptor gene

6. Which of the following is incorrect regarding the diagnosis of autoimmune myasthenia gravis?

a. Improvement in muscle weakness after administration of edrophonium suggests the diagnosis of myasthenia gravis

b. Anti-striational muscle antibodies are the most useful serologic test for the diagnosis

c. Acetylcholine receptor binding antibodies have the highest sensitivity among all serologic tests for myasthenia gravis

d. In a minority of patients with negative acetylcholine receptor binding antibodies, blocking or modulating antibodies may be positive

e. Increased jitter seen on single-fiber EMG and more than 10% decrement seen on repetitive nerve stimulation are electrophysiologic findings in disorders of the neuromuscular junction

7. A 41-year-old man presents for evaluation of weakness. On examination, there is evidence of proximal weakness and very mild facial weakness. He also has early findings suggestive of cataracts. A genetic test was ordered, and he was found to have CCTG expansion in an intron of the zinc finger protein 9 gene. Which of the following is the most likely diagnosis?

a. Becker muscular dystrophy

b. Dystrophic myotonia type 1 (DM1) myotonic dystrophy

c. Emery-Dreifuss muscular dystrophy

d. Fascioscapulohumeral muscular dystrophy

e. DM2 myotonic dystrophy

8. A baby is diagnosed with a congenital muscular dystrophy. He is floppy and weak, with contractures at the hip, knee, and ankles. As he grows, he has significant problems with cognitive and speech development, and by 18 months, he starts having seizures. The muscle biopsy shows dystrophic changes and reduced α-dystroglycan. A mutation of the fukutin gene is detected. An MRI shows white matter changes in the frontal head regions. Which of the following is the most likely diagnosis?

a. Fukuyama-type congenital muscular dystrophy

b. Laminin-α-2 deficiency

c. Bethlem myopathy

d. Muscle-eye-brain disease

e. Ullrich’s congenital muscular dystrophy

9. A 20-year-old man presents for evaluation of weakness of his face and upper extremities, gradually progressing over the past few years. The patient has a nearly expressionless face, with difficulty closing his eyes tightly and pursing his lips. He has asymmetric proximal weakness of his upper extremities to the point where he has difficulty lifting his arms above his head. He also has peroneal weakness. On examination, there is muscle atrophy, more prominent proximally in the upper extremities, with evidence of winged scapula. Interestingly, his upper arms seem “thinner” than his forearms, but his deltoids are relatively spared. Creatine kinase is 510 IU/L (normal 220 IU/L). Which of the following is the most likely diagnosis?

a. Becker muscular dystrophy

b. Duchenne muscular dystrophy

c. Emery-Dreifuss muscular dystrophy

d. Fascioscapulohumeral muscular dystrophy

e. Myotonic dystrophy

Questions 10–11

10. A 52-year-old man originally from Montreal presents to the clinic with gradually progressive dysphagia, dysphonia, and bilateral ptosis. There is no fatigability. Besides ptosis and the evident dysphonia, the rest of the neurologic examination is unremarkable. A Tensilon test is negative, as well as myasthenia gravis antibody studies. His brain MRI does not show abnormalities. Which of the following is the most likely diagnosis?

a. Becker muscular dystrophy

b. Inclusion body myositis

c. Emery-Dreifuss muscular dystrophy

d. Fascioscapulohumeral muscular dystrophy

e. Oculopharyngeal muscular dystrophy

11. Which of the following is correct regarding this condition?

a. The abnormality is caused by a GGG repeat expansion in the poly-A–binding protein gene

b. It is inherited in an X-linked recessive fashion

c. Histologically, there is variation in fiber size, rimmed vacuoles, and intranuclear tubular filaments

d. Myotonia is an evident phenomenon in this condition

e. Creatine kinase levels are elevated 10 to 100 times that of normal

12. A 25-year-old man was referred from the infertility clinic because of weakness. The patient has frontal balding, atrophy of the temporalis and masseter muscles, and weakness of the sternocleidomastoids and bilateral ptosis. He has also upper extremity weakness especially in forearm extensors, as well as atrophy in the anterior tibial muscles, with weakness of dorsiflexion of the feet. After percussion of his thenar eminence, there is prolonged contraction and slow relaxation. Which of the following is the most likely diagnosis?

a. Becker muscular dystrophy

b. Dystrophic myotonia type 1 (DM1) myotonic dystrophy

c. Emery-Dreifuss muscular dystrophy

d. Fascioscapulohumeral muscular dystrophy

e. DM2 myotonic dystrophy

13. A 44-year-old otherwise healthy man is brought to the emergency department with an episode of loss of consciousness. He had dressed up to go to his niece’s wedding and felt particularly uncomfortable in his suit and tie. He usually wore farming clothes, did not like to be formal, and felt his tie was very tight. He was leaving his house to head to the wedding when his wife called him from the kitchen. He turned his head around to see what she wanted, and suddenly, without warning, lost consciousness for a few seconds. His wife called emergency medical services, and when they arrived, his pulse was 40 bpm and his systolic blood pressure was 88 mm Hg. On arrival to the emergency department 20 minutes later, vital signs and physical examination were normal. What is the most likely diagnosis in this patient?

a. Vasovagal syncope

b. Asphyxiation from his tight tie

c. Carotid sinus hypersensitivity

d. Orthostatic hypotension with syncope

e. Severe aortic stenosis leading to syncope

14. A 10-year-old boy is brought for evaluation of muscle cramps. After exercise, the patient experiences muscle cramps, weakness, and contractures. After he rests for a few minutes, he can resume the activity. During the contractures, a needle EMG was obtained from an affected muscle, and it was electrically silent. A muscle biopsy with immunohistochemistry showed absence of myophosphorylase. Which of the following is the most likely diagnosis?

a. Tarui disease

b. McArdle’s disease

c. Cori’s disease

d. Andersen’s disease

e. Pompe’s disease

Questions 15–16

15. A 66-year-old man presents to the emergency department complaining of a 2-week history of marked slurring of speech, difficulty swallowing, and trouble holding his head up. He had been diagnosed with myasthenia gravis 2 years earlier, but symptoms had only included diplopia up until he had a flu-like illness 2 weeks earlier. On examination, he has bilateral ptosis and severe dysarthria. He cannot hold his head up off the bed when supine. He is tachypneic and is using accessory muscles to breathe. His negative inspiratory force is −15 cm H2O. What is the most appropriate next step in the management of this patient?

a. Provide him with a prescription for pyridostigmine and arrange for outpatient follow-up for him

b. Administer intravenous methylprednisolone to him immediately while he is in the ED

c. Admit him to the hospital and observe him with frequent checks of respiratory function with forced vital capacity and negative inspiratory force measurements

d. Admit him to the ICU, intubate him, and initiate therapy with intravenous immunoglobulin or plasma exchange

e. Admit him and perform emergency thymectomy

16. Regarding the management of patients with myasthenia gravis, which of the following is incorrect?

a. Thymectomy is indicated only in patients with thymoma

b. There are several medications that should be avoided as they can exacerbate the disease, including aminoglycosides and β-blockers

c. Pyridostigmine provides symptomatic relief but does not modify the course of the illness

d. Corticosteroids can cause a transient worsening of symptoms in some patients

e. Long-term immunosuppressive therapy, including azathioprine and mycophenolate mofetil, is used as steroid-sparing agents

17. An 8-year-old boy presents for evaluation of episodes of stiffness and difficulty relaxing his muscles. He has difficulty opening his eyes after closure and difficulty releasing after grasping with his hands. These symptoms are worse after exercising and performing the same task multiple times and also worsen with exposure to cold. His father also has similar symptoms. Which of the following is the most likely diagnosis?

a. Paramyotonia congenita

b. Becker’s disease

c. Thomsen’s disease

d. Hyperkalemic periodic paralysis

e. Hypokalemic periodic paralysis

18. A 32-year-old woman requires blood testing as part of a pre-employment health evaluation. She goes to the laboratory and has her blood drawn without any problems. However, she starts to feel light-headed and begins sweating profusely, her vision dims and she loses consciousness and lies listless on the floor. There is no convulsive activity or loss of bowel or bladder control. Approximately 15 seconds later she regains consciousness. She feels slightly light-headed, but is not confused and is able to recall all the events preceding her loss of consciousness. A few minutes later she is feeling fine. What is the most likely diagnosis in this patient?

a. Vasovagal syncope

b. Glossopharyngeal neuralgia

c. A seizure

d. Carotid sinus hypersensitivity

e. Third ventricular mass

Questions 19–20

19. An 8-year-old boy is brought by his parents to the clinic. His motor development has been significantly delayed, and he has difficulty walking. His weakness is predominantly in the proximal muscles, with severe weakness of hip flexors, quadriceps, gluteal, and pretibial muscles. He also has pectoral and shoulder weakness and winging of the scapulas. His calves are enlarged, with a “rubbery” texture. His intelligence quotient (IQ) is low. His serum creatine kinase is 11,560 IU/L (normal 220 IU/L). A muscle biopsy is obtained, which is shown in Figure 10.1. Which of the following is the most likely diagnosis?

FIGURE 10.1 Muscle biopsy specimen (Courtesy of Dr. Richard A. Prayson). Shown also in color plates

a. Becker muscular dystrophy

b. Duchenne muscular dystrophy

c. Emery-Dreifuss muscular dystrophy

d. Fascioscapulohumeral muscular dystrophy

e. Limb-girdle muscular dystrophy

20. Which of the following is incorrect regarding this condition?

a. It is autosomal recessive

b. Cardiac involvement occurs in these patients

c. Death commonly occurs from respiratory failure

d. The dystrophin protein is completely absent from muscles

e. Histopathologically, there is segmental degeneration and regeneration

21. A 21-year-old man from Japan is brought for evaluation of gait disturbance. He has bilateral foot drop and weakness, affecting the anterior leg compartment. There is mild weakness in the upper extremities, predominantly in the extensor muscles. Muscle biopsy shows rimmed vacuoles, and electron microscopy shows tubular filaments, resembling those seen in inclusion body myositis. His parents do not have a muscle disease. Which of the following is the most likely diagnosis?

a. Miyoshi myopathy

b. Nonaka myopathy

c. Welander myopathy

d. Markesbery-Griggs myopathy

e. Scapuloperoneal muscular dystrophy

22. A 46-year-old woman presents with diplopia occurring predominantly in the afternoon or after prolonged reading or watching television. She is diagnosed with myasthenia gravis on the basis of serologic testing, and she is prescribed pyridostigmine 60 mg, to be taken three times a day as needed for diplopia. She presents to her physician 3 days later complaining of diarrhea and crampy abdominal pain that seem to occur in the afternoon, after she has taken a dose of pyridostigmine. Which of the following is correct regarding pyridostigmine?

a. It is an acetylcholine agonist

b. It leads to destruction of acetylcholine esterase

c. Common side effects include gastrointestinal symptoms such as crampy abdominal pain and diarrhea

d. Pyridostigmine can cause significant CNS side effects, including sedation and confusion

e. Aside from gastrointestinal side effects, there is little risk of excessive dosing of pyridostigmine

23. A 5-year-old boy is brought for evaluation because of weakness and a possible muscle problem. A muscle biopsy is obtained, which is shown in Figure 10.2. Which of the following is incorrect regarding this condition?

FIGURE 10.2 Muscle biopsy specimen (Courtesy of Dr. Richard A. Prayson). Shown also in color plates

a. It is autosomal dominant, and the gene involved is RYR1

b. These patients are at risk for malignant hyperthermia

c. There is prominent proximal weakness, especially in the pelvic girdle

d. Pathologically, there are small cores that lack mitochondria, occurring only in some segments of the muscle fiber

e. Facial, bulbar, and ocular muscles are usually spared

Questions 24–25

24. A 52-year-old previously healthy woman wakes up one morning and gets up out of bed to note significant light-headedness. She makes little of it, but throughout the day she feels light-headed, and over the following few days, the symptoms progress. Two weeks after symptom onset, she can barely sit or stand because of severe light-headedness. During that time period, she began to notice early satiety, constipation, and a sensation of incomplete voiding after urination. She denies any sensory or motor symptoms in her extremities. On examination, her blood pressure is 160/90 mm Hg in the supine position with a heart rate of 75 bpm. In the seated position, her blood pressure is 120/75 mm Hg and heart rate is 78 bpm. In the standing position, her blood pressure is 90/40 mm Hg and heart rate is 83 bpm. Her examination is otherwise normal. What is the most likely diagnosis in this patient?

a. Postural orthostatic tachycardia syndrome

b. Autoimmune autonomic ganglionopathy

c. Amyloid autonomic neuropathy

d. Diabetic autonomic neuropathy

e. Pure autonomic failure

25. For the patient described in question 24, which of the following laboratory tests may be helpful in making the diagnosis?

a. Acetylcholine receptor binding antibodies

b. Ganglionic nicotinic acetylcholine receptor antibodies

c. Anti-striational muscle antibodies

d. Antibodies against presynaptic P/Q-type voltage-gated calcium channels

e. Anti-Jo 1 antibodies

26. A 20-year-old man is brought for evaluation of weakness, difficulty with plantarflexion, and atrophy of the posterior leg compartment. Genetic testing is obtained, confirming a mutation in the gene for dysferlin. His parents do not have a muscle disease. Which of the following is the most likely diagnosis?

a. Miyoshi myopathy

b. Nonaka myopathy

c. Markesbery-Griggs myopathy

d. Welander myopathy

e. Scapuloperoneal muscular dystrophy

27. A 22-year-old woman presented to the emergency department complaining of 2 weeks of dyspnea on exertion that had worsened in the prior 3 days. Over the prior 2 months, she had been experiencing difficulty swallowing, and she had noticed that holding her head up was effortful. On examination, she was dysarthric, was using accessory muscles to breath, and was holding her chin up with her hand. Neck flexor strength was Medical Research Council grade 3/5, and proximal arm muscle strength was also 3/5. Strength did not transiently improve with sustained contraction. She had no complaints of diplopia or other vision symptoms, and extraocular movements were normal. Deep tendon reflexes were also normal. She was admitted to the ICU for further evaluation and treatment. Routine laboratory testing and creatine kinase levels were normal. Serum antibodies against acetylcholine receptor were not detected on testing, but EMG showed evidence of abnormal decremental response on repetitive nerve stimulation studies. What test is likely to yield a diagnosis in this patient?

a. Antibodies against presynaptic calcium channels

b. Anti-striational muscle antibodies

c. Anti-Jo 1 antibodies

d. Anti-GQ1b antibodies

e. Anti–muscle-specific tyrosine kinase antibodies

28. Which of the following neurotransmitter-receptor pairs and their functions are incorrect?

a. Norepinephrine acts at α1-receptors on smooth muscles of blood vessels to cause vasoconstriction

b. Norepinephrine acts on β1-receptors on the sinus node to increase heart rate

c. Norepinephrine acts on β2-receptors on bronchial smooth muscles to cause bronchodilation

d. Acetylcholine acts at nicotinic receptors in the heart to reduce heart rate

e. Acetylcholine acts at M3-receptors in the salivary glands to increase secretions

29. Which of the following is not characteristic of congenital muscular dystrophies?

a. Decreased movements in utero

b. Hypotonia and weakness at birth

c. Autosomal dominant inheritance

d. Dystrophic changes on muscle biopsy

e. Frequent involvement of the brain

Questions 30–31

30. A 29-year-old man presents to the clinic for evaluation of weakness. He recalls that since he was a child, he was not able to keep up with his peers when running, biking, or in other sports, and recalls frequent falls that were attributed to clumsiness. He has proximal weakness, with difficulty standing from the sitting position, requiring the use of his arms to do so. He says he cannot lift objects above his head either. On examination, there is pseudohypertrophy of the calves. His creatine kinase is 10,490 IU/L (normal 220 IU/L). Needle EMG shows fibrillations, positive waves, and myopathic motor unit potentials. A biopsy is obtained, showing fibers of variable size, segmental degeneration and regeneration, and areas of fibrosis. The patient recalls that a maternal uncle had difficulty walking and was wheelchair-bound by age 40. Which of the following is the most likely diagnosis?

a. Becker muscular dystrophy

b. Duchenne muscular dystrophy

c. Emery-Dreifuss muscular dystrophy

d. Fascioscapulohumeral muscular dystrophy

e. Myotonic dystrophy

31. Which of the following is incorrect regarding this condition?

a. Cardiac involvement may occur

b. Intelligence quotient (IQ) is typically normal

c. The dystrophin protein is completely absent from muscles

d. It is X-linked recessive

e. The alteration of dystrophin makes the sarcolemma susceptible to rupture

Questions 32–33

32. A 40-year-old woman presents with weakness. She has difficulties standing from the sitting position and walking up the stairs. She also complains of difficulty lifting her arms above her head when she washes her hair. On examination, she has a purplish discoloration around her eyelids, and her face looks erythematous. She also has a purple scaly rash on the extensor surface of her hands, and her palmar region is thickened. Serum creatine kinase levels are elevated. A muscle biopsy is obtained, which is shown in Figure 10.3. Which of the following is the most likely diagnosis?

FIGURE 10.3 Muscle biopsy specimen (Courtesy of Dr. Richard A. Prayson). Shown also in color plates

a. Dermatomyositis

b. Polymyositis

c. Inclusion body myositis

d. Mitochondrial myopathy

e. A muscular dystrophy

33. Regarding this condition, which of the following is incorrect?

a. Methotrexate should be used, especially if there is interstitial lung disease

b. Age-appropriate malignancy work-up is indicated

c. Perifascicular atrophy is a characteristic histopathologic finding

d. It has been associated with interstitial lung disease

e. Steroids are the initial treatment of choice

34. A 62-year-old man presents with complaints of “whoozy” dizziness over the prior year. When he would wake up in the morning, while still lying in bed, he would feel fine. His wife had checked his blood pressure once in the early morning and had found it to be 180/90 mm Hg. As soon as he would get out of bed, the “whoozy” feeling would occur and would typically last throughout the day. His blood pressure during the day was as low as 80/60 mm Hg when standing. He felt particularly symptomatic after eating and when the weather was warm. He had lost consciousness twice in prior months; all episodes were triggered by changes in posture. He was otherwise relatively healthy, except for a 1-year history of impotence. He also reported frequent nocturia, urinary hesitancy, and urinary urgency. Besides demonstration of orthostasis on vital sign testing, his examination is entirely normal, with no evidence of ataxia or rigidity and normal sensory examination. What is the most likely diagnosis in this patient?

a. Multiple system atrophy

b. Autoimmune autonomic ganglionopathy

c. Familial amyloid neuropathy

d. Diabetic autonomic neuropathy

e. Pure autonomic failure

35. A 35-year-old man presents with progressive weakness in the proximal muscles of the upper and lower extremities. No skin rash is appreciated. Creatine kinase levels are elevated and a muscle biopsy is obtained, which is shown in Figure 10.4. Which of the following is incorrect?

FIGURE 10.4 Muscle biopsy specimen (Courtesy of Dr. Richard A. Prayson). Shown also in color plates

a. Steroids are the initial treatment of choice

b. An association with malignancy is not well established

c. There is an association with interstitial lung disease

d. The presence of rimmed vacuoles is very characteristic

e. It is an inflammatory myopathy

36. A 65-year-old man with diabetes, hypertension, and hyperlipidemia suffered a stroke about 1 month ago. At that time, he was started on aspirin, clopidogrel, and a statin. He now presents with generalized weakness, myalgias, shortness of breath, and oliguria. He seems to be fluid overloaded, and his blood urea nitrogen and creatinine are very elevated. His creatine kinase (CK) is 15,460 IU/L (normal up to 220 IU/L). Which of the following is incorrect?

a. Statins can produce rhabdomyolysis

b. SLCO1B gene mutation is associated with predisposition to statin-induced myopathy

c. Statins can produce myalgias without increased serum CK

d. Statins can produce asymptomatic increases in serum CK

e. Concomitant intake of statins and fibrates does not increase the risk of muscle toxicity

37. A 25-year-old woman with depression, migraines, and chronic fatigue syndrome presents to the clinic complaining of palpitations and light-headedness every time she stands from a seated position. These symptoms never occur while seated or supine. On vital sign testing, her pulse increases from 90-bpm while seated to 130 bpm while standing. Her examination is otherwise entirely normal. Tilt table testing confirms an increase of pulse from 80 bpm when supine to 150 bpm with head-up tilt (at approximately 70 degrees); blood pressure remains stable during the tilt. With the increase in heart rate she feels palpitations, light-headedness, and generalized weakness, without flushing or skin changes. Laboratory evaluation, echocardiography, and other testing are otherwise normal. What is the most likely diagnosis in this patient?

a. Orthostatic hypotension

b. Vasovagal response

c. Systemic mastocytosis

d. Postural orthostatic tachycardia syndrome

e. Subclavian steal syndrome

38. A 45-year-old man originally from China has a history of palpitations and anxiety and presents with recent onset of episodes of weakness affecting arms and legs to the point that he feels paralyzed. There is sparing of respiratory and bulbar muscles. Which of the following should be done next?

a. EMG needle examination

b. Send genetic testing for SCN4A mutation

c. Refer to psychiatry

d. Obtain thyroid-stimulating hormone

e. Begin carbonic anhydrase inhibitors

Questions 39–40

39. A 59-year-old man presents with asymmetric atrophy and weakness of the wrist and finger flexor muscles, as well as the quadriceps femoris. There is also weakness of anterior tibial muscles. The deltoids are spared. Serum creatine kinase levels are slightly elevated. The muscle biopsy is shown in Figure 10.5. Which of the following is the most likely diagnosis?

FIGURE 10.5 Muscle biopsy specimen (Courtesy of Dr. Richard A. Prayson). Shown also in color plates

a. Dermatomyositis

b. Polymyositis

c. Inclusion body myositis

d. Mitochondrial myopathy

e. A muscular dystrophy

40. Regarding the condition depicted in question 39, which of the following is correct?

a. Rimmed vacuoles are characteristic of this condition

b. There is perifascicular atrophy, but no inflammatory cells within the fascicles

c. Gottron’s papules are characteristic

d. “Mechanic hands” are usually seen in these patients

e. This condition responds well to prednisone

41. Which of the following is incorrect regarding the limb-girdle muscular dystrophies (LGMD)?

a. There is no cardiac involvement in any type of LGMD

b. Muscle biopsy shows dystrophic changes

c. Needle EMG shows myopathic changes

d. Some types are associated with contractures

e. The LGMD1 group is autosomal dominant

42. A 10-year-old boy is brought for evaluation of proximal weakness, affecting predominantly the pelvic girdle. A limb-girdle muscular dystrophy (LGMD) is suspected. Which of the following proteins is associated with its respective type of LGMD?

a. Myotilin and LGMD2A

b. Caveolin-3 and LGMD2B

c. Dysferlin and LGMD1A

d. Fukutin-related protein and LGMD2I

e. Sarcoglycan and LGMD1C

Questions 43–44

43. A 52-year-old woman who has smoked since adolescence presented to the clinic complaining of weakness in her arms that she had noticed while blow-drying her hair and weakness in her legs when climbing stairs. Associated symptoms included constipation and a sensation of incomplete emptying of the bladder. On examination, she had Medical Research Council grade 4/5 weakness in proximal arm muscles and in hip flexors. Deep tendon reflexes were 1+ throughout. After brief sustained contraction of her biceps muscle, the biceps deep tendon reflexes increased to 2+. Medial nerve CMAPs are shown in Figure 10.6. EMG did not show evidence of denervation or myopathy. Serum creatine kinase was normal. What is the most likely diagnosis in this patient?

FIGURE 10.6 Median nerve CMAP tracings (Courtesy of Dr. Robert Shields)

a. Autoimmune myasthenia gravis

b. Paraneoplastic Lambert-Eaton myasthenic syndrome

c. Polymyositis

d. Inclusion body myositis

e. Acute demyelinating polyneuropathy

44. Regarding the condition described in question 43, which of the following is incorrect?

a. It is due to antibodies against presynaptic P/Q-type voltage-gated calcium channels

b. It can be either paraneoplastic or autoimmune

c. It is more common in males

d. Autonomic nervous system involvement occurs

e. Symptoms respond well to acetylcholine esterase inhibitors

45. A 42-year-old man who was previously healthy presents with 1 week of puffy eyelids, diplopia, dysphagia, myalgias, and proximal greater than distal muscle weakness. Creatine kinase levels are only mildly elevated, but the serum eosinophil count is markedly increased. A muscle biopsy is obtained, which is shown in Figure 10.7. Which of the following is the most likely diagnosis?

FIGURE 10.7 Muscle specimen (Courtesy of Dr. Richard A. Prayson). Shown in color plates

a. Dermatomyositis

b. Polymyositis

c. Inclusion body myositis

d. Trichinosis

e. Muscular dystrophy

46. A baby is brought for evaluation of hypotonia, and on muscle biopsy, there are sarcoplasmic rods consistent with nemaline myopathy. Which of the following is incorrect regarding this condition?

a. There is great variability in age of onset, with presentations in the neonatal period as well as in adults

b. This condition can be inherited in an autosomal dominant or recessive fashion

c. Respiratory muscles are rarely affected in adult forms

d. α-actin, α-tropomyosin, and β-tropomyosin are genes implicated

e. Nebulin, troponin, and cofilin are genes implicated

47. Regarding testing of the autonomic nervous system, which of the following statements is incorrect?

a. Tilt table testing is a measure of autonomic function and assesses changes in blood pressure and heart rate that occur with changes in posture

b. Normally, with assumption of the upright posture, there is transient bradycardia and hypotension followed by blood pressure and heart rate normalization

c. In the evaluation of syncope, with upright tilt table testing, an abrupt reduction in blood pressure with bradycardia signifies a neurocardiogenic mechanism

d. The thermoregulatory sweat test is a qualitative test of sudomotor function, which can identify patterns of sweating abnormalities that may correspond to different forms of dysautonomia

e. The quantitative sudomotor axon reflex test is a measure of sudomotor function and specifically assesses the sympathetic postganglionic axon

Questions 48–49

48. A 1-month-old baby is evaluated for difficulty feeding and cyanosis. He is hypotonic and weak, and is noticed to have macroglossia, cardiomegaly, and hepatomegaly. A diagnosis of Pompe’s disease is made. Which of the following is incorrect?

a. It is caused by acid maltase deficiency

b. There is glycogen accumulation in the affected tissues

c. This is type V glycogenosis

d. It is autosomal recessive

e. It is caused by deficiency of lysosomal α-1,4-glucosidase

49. A 29-year-old man has had a slowly progressive decline in exercise tolerance over the past year. The patient is currently in the ICU intubated for respiratory failure that started 2 days ago after acquiring an upper respiratory infection. An adult form of acid maltase deficiency is suspected. Which of the following is correct if this is the case?

a. Given the age of onset and the lack of cardiac and liver involvement, this patient does not have acid maltase deficiency

b. Muscle biopsy with vacuolated sarcoplasm with glycogen accumulation that stains strongly with acid phosphatase helps making the diagnosis of acid maltase deficiency

c. This patient has type III glycogenosis

d. Acid maltase deficiency is autosomal dominant

e. Mental retardation is common with presentations in this age group

50. Consultation is requested for a 6-month-old baby who in the neonatal period had required prolonged mechanical ventilation, tracheostomy, and percutaneous gastrostomy insertion. On examination, he has bilateral ptosis, and restricted eye movements are noted. Pupils are equal and briskly react to light. His mother is healthy, with no evidence of a neuromuscular disorder. On administration of edrophonium, his ptosis improves, his sucking reflex becomes stronger, and his eye movements achieve full range. What is the most likely diagnosis in this patient?

a. Transient neonatal myasthenia

b. Botulism

c. Congenital myasthenia due to congenital acetylcholine receptor deficiency

d. Congenital myasthenia due to choline acetyltransferase deficiency

e. A mitochondrial disorder

Questions 51–52

51. A 14-year-old boy has progressive weakness predominantly of the shoulders and upper arms, with milder involvement of the lower extremities. He has prominent contractures at the elbows and ankles, but no pseudohypertrophy of the calves. An electrocardiogram was obtained because he was found to be bradycardic, and a complete atrioventricular block was detected. Which of the following is the most likely diagnosis?

a. Becker muscular dystrophy

b. Duchenne muscular dystrophy

c. Emery-Dreifuss muscular dystrophy

d. Fascioscapulohumeral muscular dystrophy

e. Limb-girdle muscular dystrophy

52. Which of the following is/are the defective protein(s) in this condition?

a. Emerin

b. Laminin A/C

c. Dystrophin

d. a and b

e. a and c

53. A baby is diagnosed with a congenital muscular dystrophy. Clinically, he is weak and has multiple contractures with distal hyperlaxity. There are protrusions of the calcanei in the feet. Which is the most likely diagnosis?

a. Fukuyama-type congenital muscular dystrophy

b. Laminin-α-2 deficiency

c. Walker-Marburg syndrome

d. Muscle-eye-brain disease

e. Ullrich’s congenital muscular dystrophy

54. Which of the following is correct regarding the anatomy of the sympathetic and parasympathetic systems?

a. Parasympathetic postganglionic neurons predominantly release norepinephrine

b. Cell bodies of neurons that provide innervation to the detrusor muscle of the bladder are located in L2 to L4

c. The intermediolateral cell column is present from T1 to L5 and is the source for preganglionic sympathetic fibers

d. Gray rami communicantes carry preganglionic fibers, and white rami communicantes carry postganglionic fibers

e. Parasympathetic ganglia are located close to the end organ, whereas sympathetic ganglia are distant from their end organ; sympathetic postganglionic fibers are longer as compared to parasympathetic postganglionic fibers

Questions 55–56

55. A 62-year-old woman with diabetes who has smoked since her teenage years is being evaluated during her annual examination. She has been doing relatively well except for a chronic cough, which she attributes to smoking. She fell a few days ago, but did not hit her head. During examination, her physician notices that her left eyelid is droopy. On further examination, in dim light, her left pupil is noted to be smaller than the right. The patient’s left face appears to be clearly less moist as compared to the right and slightly paler. Extraocular movements are normal. What is the most likely diagnosis in this patient?

a. An ICA dissection that she probably sustained when she fell

b. An oculomotor palsy due to a lacunar infarct

c. Ptosis due to levator dehiscence

d. A partial third nerve palsy due to diabetic cranial neuropathy

e. Horner’s syndrome due to an apical lung mass (Pancoast tumor)

56. A 42-year-old man is involved in a motor vehicle accident. He experiences right-sided neck pain for 2 days following the accident and presents to his physician for further evaluation. On examination, he has drooping of his right eyelid. The right pupil is smaller than the left, and his right eye appears sunken. A carotid ultrasound is performed, and shows a right ICA dissection with an intact right ECA. Which of the following statements is correct?

a. In this patient, there will be anhidrosis on the right side of the face

b. His eyelid is droopy because of involvement of the levator palpebrae

c. The eye abnormalities have resulted from extension of the dissection into the cavernous portion of the carotid artery, leading to involvement of cranial nerve III

d. This patient’s anisocoria will become more apparent in dim light

e. This patient is likely to have reduced abduction of the right eye on attempted gaze to the right

57. Which of the following is not a distal muscular dystrophy?

a. Dystrophic myotonia type 2 (DM2) myotonic dystrophy

b. Miyoshi myopathy

c. Welander myopathy

d. Desmin myopathy

e. Nonaka myopathy

Questions 58–60

58. Which of the following statements is incorrect regarding innervation of the genitourinary system?

a. The intermediolateral cell column in the spinal cord at the level of L1 and L2 provides sympathetic innervation to the bladder

b. Nerve roots S2 to S4 relay afferent sensory information from the genitourinary system, bladder, and anorectal area to the spinal cord

c. Somatic efferents to the skeletal muscles of the pelvic floor arise from the anterior horn cells at S2 to S4 and are carried by the pudendal nerves

d. Onuf’s nucleus at S2 to S4 contains the cell bodies of neurons that control the urethral and anal sphincters

e. Penile erection is mediated by the sympathetic nervous system and ejaculation by the parasympathetic nervous system

59. Which of the following pairs of urinary symptoms and associated bladder abnormalities are incorrect in the patients described?

a. A 62-year-old man with cauda equina syndrome who involuntarily urinates when pressure is applied to his bladder—flaccid areflexic bladder leading to overflow incontinence

b. A 72-year-old man with normal pressure hydrocephalus who has no control over urination—dysfunction of the medial frontal micturition centers in the paracentral lobule, leading to loss of voluntary suppression of the detrusor reflex

c. A 17-year-old male who sustained a thoracic cord contusion 2 days ago and is unable to urinate—atonic bladder with reflex contraction of the urethral sphincter, leading to urinary retention

d. A 32-year-old woman with multiple sclerosis who has multiple spinal cord lesions and sometimes has incontinence because she senses the need to urinate but has such urgency that she cannot always make it to the bathroom in time—detrusor hyperreflexia

e. A 54-year-old man with multiple sclerosis who has severe painful pelvic cramps associated with a sensation that he needs to urinate, but after urination, feels he has not voided completely—detrusor-sphincter dyssynergia resulting from contraction of the detrusor muscle with excessive relaxation of the internal and external urethral sphincter

60. Regarding bladder function, which of the following statements is incorrect?

a. There is normally voluntary inhibition of the detrusor reflex by the medial frontal micturition center located in the paracentral lobule

b. External urethral sphincter tone is normally under voluntary control

c. The internal urethral sphincter is under parasympathetic control

d. The micturition center, located in the pons, regulates the detrusor reflex

e. In detrusor-sphincter dyssynergia, the detrusor muscle contracts against an unrelaxed urethral sphincter

61. A 3-week-old child is brought to a pediatrician because of distended abdomen. His mother reports that he has had a few bowel movements, but that he was certainly not having the normal bowel movements that her other children at his age. On examination, he has a distended abdomen with hypoactive bowel sounds. X-ray of the abdomen shows dilated loops of bowel with excessive fecal material. A diagnosis of Hirschsprung’s disease is suspected after additional testing. Which of the following is incorrect regarding the enteric nervous system?

a. It consists of the myenteric plexus, located between the outer and inner smooth muscle layers of the gastrointestinal tract, and the submucous plexus, located between the circular muscle layer and the mucosa

b. The myenteric plexus is predominantly involved in gut motility

c. The submucous plexus is predominantly involved in secretory functions of the gastrointestinal tract

d. Hirschsprung’s disease is due to congenital absence of the submucous plexus

e. Hirschsprung’s disease is often focal, though rare cases involve the entire colon

62. Which of the following is incorrect regarding myofibrillar myopathy?

a. Desmin is a protein implicated

b. αβ-Crystallin is one of the proteins implicated

c. There is no cardiac involvement

d. Peripheral neuropathy can be seen in these patients

e. There is focal dissolution of myofibrils and subsarcolemmal accumulation of dense granular and filamentous material

63. A 6-year-old boy is brought for evaluation of stiffness. His legs are stiff, and he has difficulty relaxing muscles after contracting them. For example, he cannot release objects easily once grasped with his hands, and when he closes his eyes, it takes a few seconds before he is able to open them completely. If he performs the same motor task multiple times, it becomes easier. When his thenar eminence is percussed, there is prolonged contraction and delayed relaxation. His father and grandfather have similar clinical features that started around the same age. Which of the following is the most likely diagnosis?

a. Paramyotonia congenita

b. Myotonia congenita, Becker’ disease

c. Myotonia congenita, Thomsen’s disease

d. Hyperkalemic periodic paralysis

e. Hypokalemic periodic paralysis

64. A 39-year-old man with a history of alcoholism is admitted with pancreatitis and significant hyperglycemia. His hospital course is complicated by acute respiratory distress syndrome and sepsis. He requires prolonged intubation, and because he “fights” the ventilator, he required sedation and paralysis. Tracheostomy is required. After 3 weeks of being in the ICU, the neurology team is called because the patient does not move any of his limbs and his limbs are flaccid. NCS show that SNAPs are normal, with low CMAPs. Needle EMG study shows myopathic findings. Which of the following is incorrect?

a. There is myosin loss

b. This patient has critical illness polyneuropathy

c. Systemic inflammatory response syndrome is associated with this condition

d. Steroids and neuromuscular blocking agents are risk factors for this illness

e. This is a potential cause for difficulty weaning patients from mechanical ventilation

65. A 10-year-old boy is brought for evaluation because of contractures. He has flexion contractures of the elbows and ankles, as well as hyperextensible interphalangeal joints. His father apparently also had contractures since adolescence. A diagnostic test demonstrates mutations associated with the gene that encodes collagen type VI. Which is the most likely diagnosis?

a. Fukuyama-type congenital muscular dystrophy

b. Laminin-α-2 deficiency

c. Bethlem myopathy

d. Muscle-eye-brain disease

e. Walker-Marburg syndrome

Answer Key

1. b

2. e

3. d

4. b

5. c

6. b

7. e

8. a

9. d

10. e

11. c

12. b

13. c

14. b

15. d

16. a

17. a

18. a

19. b

20. a

21. b

22. c

23. d

24. b

25. b

26. a

27. e

28. d

29. c

30. a

31. c

32. a

33. a

34. e

35. d

36. e

37. d

38. d

39. c

40. a

41. a

42. d

43. b

44. e

45. d

46. c

47. b

48. c

49. b

50. c

51. c

52. d

53. e

54. e

55. e

56. d

57. a

58. e

59. e

60. c

61. d

62. c

63. c

64. b

65. c

Answers

1. b

This patient has hypokalemic periodic paralysis, which is inherited in an autosomal dominant fashion; however, sporadic cases have been reported. There are two types: type 1 caused by a mutation in the calcium channel gene CACNA1S on chromosome 1q31, and type 2 caused by a mutation in the sodium channel gene SCN4A. These patients present with episodes of weakness without myotonia that could be focal or generalized and ranges from mild to severe, associated with hyporeflexia during the attacks, each of which can last for hours, with some persistent mild weakness for a few days. During attacks, the creatine kinase level may be elevated and the serum potassium is usually reduced. These episodes are triggered by exercise, meals rich in carbohydrates, ethanol, cold exposure, and emotional stressors. Provocative testing can be done with glucose administration. The main treatment focus is to avoid triggers; carbonic anhydrase inhibitors such as acetazolamide and potassium-sparing diuretics are useful in treating this condition.

Hyperkalemic periodic paralysis, on the other hand, presents with episodes of weakness triggered by resting after exercise and fasting. This condition is autosomal dominant and is caused by a mutation in the sodium channel gene SCN4A. Provocative testing is done with administration of potassium. The treatment is focused on avoiding triggers. During attacks, glucose can be provided, and as prophylactic therapy, thiazide diuretics can be used.

Andersen-Tawil syndrome is also a channelopathy characterized by periodic paralysis, ventricular arrhythmias, and dysmorphic features. It is associated with a mutation in the potassium channel gene KCNJ2.

Paramyotonia congenita is discussed in question 17. Myotonia congenita is discussed in question 63.

Saperstein DS. Muscle channelopathies. Semin Neurol. 2008; 28:260–269.

2. e

This patient has steroid-induced myopathy, in which the needle EMG is nonspecific and creatine kinase levels are usually normal.

Steroid-induced myopathy can be caused by chronic use of exogenous corticosteroids used for the treatment of underlying inflammatory conditions or by endogenous hypercortisolism, such as in Cushing’s disease. The weakness is typically mild or moderate and proximal. Histopathologically, there is atrophy of type II fibers. Management of this condition involves treatment of the underlying cause in case of endogenous hypercortisolism or reduction of exogenous steroids, as well as physical therapy.

Dalakas MC. Toxic and drug-induced myopathies. J Neurol Neurosurg Psychiatry. 2009; 80:832–838.

3. d

Centronuclear myopathy (also known as myotubular myopathy) is a congenital myopathy, manifesting characteristically with ptosis and ocular palsies, as well as weakness of facial, pharyngeal, laryngeal, and neck muscles. It typically presents with hypotonia and weakness at birth, or in early childhood. Proximal and distal weakness and hyporeflexia can be seen. Severe forms may be fatal due to respiratory failure in the first few months of life. Creatine kinase levels are mildly elevated. Needle EMG shows a myopathic pattern with positive waves and fibrillations. Pathologically, there are small muscle fibers and central nucleation, as well as predominance of type I fibers, which are small and hypotrophic.

This condition is inherited in an X-linked or autosomal dominant or recessive fashion. The X-linked form is severe and presents in the neonatal period, the autosomal dominant form is an adult-onset milder form, and the autosomal recessive form is intermediate in severity.

Cardamone M, Darras BT, Ryan MM. Inherited myopathies and muscular dystrophies. Semin Neurol. 2008; 28:250–259.

Ropper AH, Samuels MA. Adams and Victor’s Principles of Neurology, 9th ed. New York: McGraw-Hill; 2009.

4. b

This patient has myasthenia gravis. As discussed below, the majority of patients with myasthenia gravis have generalized manifestations (not restricted to a single muscle group) within 2 years of symptom onset.

Myasthenia gravis is a neuromuscular junction transmission disorder. It is most commonly autoimmune in etiology, though there are rare cases of congenital myasthenia that are genetic (see question 50). It is more common in males and older adults. Women have an earlier age of onset, often presenting in the second or third decades of life, whereas men typically present in the sixth decade. Myasthenia gravis may present with a myriad of symptoms; the hallmarks are fluctuations and fatigability, with weakness worsening with increased muscle use. A proportion of patients present with predominantly extraocular muscle involvement, with diplopia and ptosis. The majority of such patients generalize to involve bulbar, limb, neck, and/or respiratory muscles within 2 years, though in a minority, involvement remains restricted to the eyes, so-called ocular myasthenia. Other patients present with predominantly bulbar symptoms due to involvement of muscles of mastication, speech, swallowing, and facial expression, leading to weakness and/or fatigability of chewing, dysphagia, and/or dysarthria. Limb and neck weakness also occur. Myasthenic crisis, or respiratory failure due to involvement of respiratory muscles, can be the presenting symptom, but more commonly occurs in those with exacerbation of symptoms in the setting of stressors such as infection or surgery. Myasthenia gravis is more common in family members of patients with this disorder. It has been associated with human leukocyte antigen types B8, DR1, DR2, and DR3. Patients with myasthenia gravis are at an increased risk of other autoimmune disorders including thyroid disorders, and these should be tested for as clinically indicated.

Bradley WG, Daroff RB, Fenichel GM, et al. Neurology in Clinical Practice, 5th ed. Philadelphia, PA: Elsevier; 2008.

5. c

The pathophysiology of autoimmune myasthenia gravis relates to the presence of circulating anti-acetylcholine receptor antibodies, which bind to the acetylcholine receptor causing immune-mediated destruction of the junctional folds that contain dense concentrations of this receptor, and a higher rate of internalization and destruction of acetylcholine receptor. In some cases, the antibodies might block binding of acetylcholine to its receptor at the neuromuscular junction. Antibodies against presynaptic voltage-gated calcium channels occur in Lambert-Eaton myasthenic syndrome (discussed in questions 43 and 44). Botulinum toxin inhibits exocytosis of presynaptic vesicles containing acetylcholine (discussed in Chapter 17). Rarely, myasthenia results from mutations in the acetylcholine receptor gene (discussed in question 50).

Bradley WG, Daroff RB, Fenichel GM, et al. Neurology in Clinical Practice, 5th ed. Philadelphia, PA: Elsevier; 2008.

6. b

Anti-striational muscle antibodies can be positive in myasthenia patients with thymoma, but they are much less sensitive and specific than acetylcholine receptor antibodies for the diagnosis of myasthenia gravis.

Edrophonium is an intravenous acetylcholine esterase inhibitor that increases the presence of acetylcholine at the neuromuscular junction. In patients with myasthenia, administration of edrophonium leads to transient improvement of weakness within minutes of administration; the edrophonium (Tensilon) test has therefore been used in the diagnosis of myasthenia gravis.

Laboratory testing for serum autoantibodies also aids in the diagnosis; positive tests confirm the diagnosis of autoimmune myasthenia gravis in the appropriate clinical setting, but their absence does not exclude it. Acetylcholine receptor binding antibodies can be detected by measuring binding to purified acetylcholine receptors radiolabeled with α-bungarotoxin. The sensitivity of this test is highest for generalized myasthenia, detecting antibodies in 70% to 95% of patients and lower for patients with ocular myasthenia. Patients who are initially seronegative for binding, blocking, or modulating antibodies may seroconvert later in the course of their disease. In a minority of patients with autoimmune myasthenia in whom the binding antibody is not detectable, modulating or blocking antibodies may be present. False-positive testing for these antibodies is rare, but can occur in patients with other autoimmune disorders. The patient’s clinical picture, rather than antibody levels, is used to monitor response to therapy.

In approximately half of all patients who are seronegative for antibodies against the acetylcholine receptor, anti-muscle-specific tyrosine kinase antibodies are present (discussed in question 27).

EMG findings in myasthenia gravis include electrodecremental response and jitter. The presence of a 10% or greater decrement in amplitude of a CMAP between the first and fourth to fifth stimuli with repetitive nerve stimulation suggests a neuromuscular junction (NMJ) disorder (see Chapter 9). If there is not an abnormality present on repetitive nerve stimulation but there is a high clinical suspicion for myasthenia gravis and it cannot be confirmed with serologic testing, a single-fiber EMG can be performed. Single-fiber EMG is the most sensitive test of NMJ transmission. Jitter is the variability in the measure of interpotential difference between two muscle fiber action potentials during consecutive discharges of the same motor unit. Increased jitter (or increased interpotential time) is present in patients with NMJ abnormalities, but is not specific for myasthenia gravis. Blocking on single-fiber EMG is failure of a single muscle fiber action potential to appear during the motor unit discharge, and occurs when there is significantly increased jitter.

Bradley WG, Daroff RB, Fenichel GM, et al. Neurology in Clinical Practice, 5th ed. Philadelphia, PA: Elsevier; 2008.

7. e

This patient has Dystrophic myotonia type 2 (DM2) myotonic dystrophy. This condition is also known as PROMM or proximal myotonic myopathy. It is autosomal dominant and is characterized by proximal muscle weakness and myotonia. Cataracts and cardiac involvement are less frequent. Histopathologically, there are nonspecific findings of myopathy. The cause is a CCTG repeat expansion in an intron of the zinc finger protein 9 gene on chromosome 3q, and it is associated with intranuclear accumulation of the expanded RNA transcripts.

Becker muscular dystrophy is discussed in questions 30 and 31. DM1 myotonic dystrophy is discussed in question 12. Emery-Dreifuss muscular dystrophy is discussed in questions 51 and 52. Fascioscapulohumeral muscular dystrophy is discussed in question 9.

Cardamone M, Darras BT, Ryan MM. Inherited myopathies and muscular dystrophies. Semin Neurol. 2008; 28:250–259.

Ropper AH, Samuels MA. Adams and Victor’s Principles of Neurology, 9th ed. New York: McGraw-Hill; 2009.

8. a

This patient has Fukuyama-type congenital muscular dystrophy. This condition is autosomal recessive, caused by a mutation of the fukutin gene on chromosome 9q. It is characterized by weakness and ocular and CNS abnormalities. Patients are hypotonic and floppy, with joint contractures at the hip, knee, and ankles. The weakness may be generalized, and these patients typically do not learn to walk. Creatine kinase levels are elevated, and muscle biopsy shows dystrophic changes and reduced α-dystroglycan. There is mental retardation and seizures are frequent. Brain MRI shows abnormalities in gyration and characteristic white matter changes in the frontal regions.

Laminin-α-2 deficiency is another muscular dystrophy, also known as merosin deficiency. These patients are hypotonic at birth, and have severe weakness of the trunk and limbs. Extraocular and facial muscles are spared. Contractures appear in the feet and hips. Some patients may have seizures; however, intelligence is generally preserved. MRI shows white matter changes and sometimes cortical abnormalities.

Ullrich myopathy and muscle-eye-brain disease are discussed in question 53. Bethlem myopathy is discussed in question 65.

Bradley WG, Daroff RB, Fenichel GM, et al. Neurology in Clinical Practice, 5th ed. Philadelphia, PA: Elsevier; 2008.

Cardamone M, Darras BT, Ryan MM. Inherited myopathies and muscular dystrophies. Semin Neurol. 2008; 28:250–259.

Ropper AH, Samuels MA. Adams and Victor’s Principles of Neurology, 9th ed. New York: McGraw-Hill; 2009.

9. d

This patient has fascioscapulohumeral muscular dystrophy (FSHD). This condition is inherited in an autosomal dominant fashion and is caused by deletions in a 3.3 kb repeating sequences, termed D4Z4, located on chromosome 4q35.

FSHD is slowly progressive, and predominantly affects the face and shoulders, though later in the course of the disease, the lower extremities are affected as well. Age of onset is on average 16 in males and 20 in females, but can be variable, ranging between the first and sixth decades of life. These patients have weakness that can be asymmetric, and present with difficulty lifting their arms above their head, with prominent involvement of the upper arms (scapular muscles, biceps, triceps, trapezius, serratus anterior, and pectoralis), with relative sparing of the deltoids. The upper arm seems to be more atrophic than the forearms, making the bones of the shoulder appear prominent. Facial weakness is evident, with weakness of the orbicularis oculi, zygomaticus, and orbicularis oris. There is also weakness of the lower abdominal muscles, producing the “Beevor sign,” in which the umbilicus moves upward with neck flexion. Pelvic muscles are compromised later, and involvement of peroneal muscles will manifest as foot drop. The masseters, temporalis muscle, extraocular muscles, and pharyngeal and respiratory muscles are usually spared. Cardiac involvement is rare and intelligence is typically normal. Creatine kinase levels are normal to slightly elevated.

The other options are unlikely in this case. Duchenne muscular dystrophy is discussed in questions 19 and 20. Becker muscular dystrophy is discussed in questions 30 and 31. Emery-Dreifuss muscular dystrophy is discussed in questions 51 and 52. Myotonic dystrophy is discussed in questions 7 and 12.

Bradley WG, Daroff RB, Fenichel GM, et al. Neurology in Clinical Practice, 5th ed. Philadelphia, PA: Elsevier; 2008.

Cardamone M, Darras BT, Ryan MM. Inherited myopathies and muscular dystrophies. Semin Neurol. 2008; 28:250–259.

Ropper AH, Samuels MA. Adams and Victor’s Principles of Neurology, 9th ed. New York: McGraw-Hill; 2009.

10. e, 11. c

This patient has oculopharyngeal muscular dystrophy. This is an autosomal dominant late-onset muscular dystrophy with manifestations restricted to the ocular and pharyngeal regions. It is more frequent in patients with French-Canadian inheritance, and is caused by a GCG repeat expansion in the poly-A–binding protein 2 gene on chromosome 14q11. Patients present with dysphagia, dysphonia, and slowly progressive ptosis, and sometimes late involvement of extraocular muscles. There is no myotonia. Creatine kinase and aldolase levels are normal, and EMG is abnormal in affected muscles. Muscle biopsy demonstrates variation in fiber size, rimmed vacuoles, and intranuclear tubular filaments.

Cardamone M, Darras BT, Ryan MM. Inherited myopathies and muscular dystrophies. Semin Neurol. 2008; 28:250–259.

Ropper AH, Samuels MA. Adams and Victor’s Principles of Neurology, 9th ed. New York: McGraw-Hill; 2009.

12. b

This patient has dystrophic myotonia type 1 (DM1) myotonic dystrophy, which is an autosomal dominant condition with high penetrance, caused by a CTG expansion in the myotonic dystrophy protein kinase gene on chromosome 19q. These patients present in early adult life with ptosis and facial weakness, and characteristic features such as frontal balding, atrophy of the masseters and temporalis, and weakness and atrophy of small muscles of the hands and extensors of the forearms and peroneal muscles. Pharyngeal and laryngeal weakness may be present, but this finding is rare. Patients have myotonia, which is a phenomenon of prolonged contraction and slow relaxation. Patients may also have diaphragmatic weakness, leading to respiratory failure, and cardiac abnormalities, especially conduction defects.

Various other tissues are also involved, and patients may exhibit esophageal dilatation, megacolon, lenticular opacities and cataracts, mental retardation, testicular atrophy with infertility, and androgen deficiency. Creatine kinase levels may be slightly elevated. EMG demonstrates myotonic discharges and short, rapidly recruiting motor unit potentials with fibrillations. Histopathologically, muscle biopsy demonstrates marked central nucleation, type 1 fiber atrophy, peripherally placed sarcoplasmic masses, ring fibers, and pyknotic nuclear clumps.

The other options are less likely to be the diagnosis in this patient. Becker muscular dystrophy is discussed in questions 30 and 31. Emery-Dreifuss muscular dystrophy is discussed in questions 51 and 52. Fascioscapulohumeral muscular dystrophy is discussed in question 9. DM2 myotonic dystrophy is discussed in question 7.

Cardamone M, Darras BT, Ryan MM. Inherited myopathies and muscular dystrophies. Semin Neurol. 2008; 28:250–259.

Ropper AH, Samuels MA. Adams and Victor’s Principles of Neurology, 9th ed. New York: McGraw-Hill; 2009.

13. c

This patient most likely has carotid sinus hypersensitivity, as suggested by the triggering of symptoms with head movement in the setting of a tight collar. Carotid sinus hypersensitivity is defined by the occurrence of syncope associated with either a period of asystole of at least 3 seconds or a fall of at least 50 mm Hg in systolic blood pressure, or both, in response to pressure on the carotid sinus. In another form of carotid sinus hypersensitivity, hypotension without bradycardia may occur. Triggers can include a tight collar, turning of the head, or even swallowing, though no identifiable triggers may be present. Symptoms and changes in heart rate and/or blood pressure can be reproduced with carotid sinus massage. Treatment is avoidance of triggers, but when recurrent, pacemaker insertion may be necessary.

The carotid sinus, located at the bifurcation of the common carotid artery, is innervated by a branch of the glossopharyngeal nerve. The carotid sinus contains specialized mechanoreceptors that are capable of detecting changes in blood pressure (baroreceptors) and heart rate. With simulation, either mechanically (with applied pressure) or with elevations in blood pressure, mechanoreceptors send signals to the nucleus tractus solitarius in the medulla leading to a reduction in sympathetic tone (leading to a further reduction in blood pressure) and an increase in parasympathetic tone (leading to a reduction in heart rate). Carotid sinus hypersensitivity results from an exaggerated response to baroreceptor stimulation.

Vasovagal syncope is discussed in question 18; the absence of diaphoresis and nausea, and the presence of bradycardia, hypotension, and triggering by neck turning distinguish the two. Orthostatic hypotension, as most commonly occurs from dehydration leading to volume depletion and subsequent reduction in blood pressure with changes in posture, is usually preceded by warning symptoms such as light-headedness, which were not present in this case. Severe aortic stenosis can lead to syncope, but bradycardia would not be present, and other associated symptoms would include chest pain and shortness of breath.

Bradley WG, Daroff RB, Fenichel GM, et al. Neurology in Clinical Practice, 5th ed. Philadelphia, PA: Elsevier; 2008.

Morhman DE, Heller LJ. Cardiovascular Physiology, 6th ed. New York: McGraw-Hill; 2006.

14. b

This patient has McArdle’s disease, or glycogenosis type V. This condition is an autosomal recessive disorder caused by myophosphorylase deficiency. This enzyme normally participates in the conversion of glycogen into glucose-6-phosphate; therefore, its deficiency will lead to glycogen accumulation and lack of glucose release from glycogen. The typical presentation is exercise-induced weakness, muscle cramps, and contractures, with electrical silence when needle EMG is performed in contractured muscles. Unlike normal muscles, when the muscle is exercised there is no production of lactic acid.

On exertion, a sensation of fatigue may ensue; however, if the patient slows down but does not stop, this sensation may disappear and the patient may be able to continue with the exercise. This is called a “second-wind phenomenon,” which is typically seen in McArdle’s disease, and may be related to a change in the blood supply, change in the muscle metabolism, and rise in fatty acid use.

Tarui disease (glycogenosis type VII) is caused by phosphofructokinase deficiency. This enzyme participates in the conversion of glucose-6-phosphate into glucose-1-phosphate, and therefore, it is similar to McArdle’s disease from the clinical standpoint. Immunohistochemical analysis distinguishes these two disorders.

Cori’s disease (glycogenosis type III) is caused by a deficiency in the debranching enzyme, leading to glycogen accumulation. These patients can present with a childhood form with liver disease and weakness or with an adult form characterized by myopathic weakness.

Andersen’s disease (glycogenosis type IV) is caused by a deficiency in the branching enzyme, and is characterized by hepatomegaly from polysaccharide accumulation, cirrhosis, and liver failure. Pompe’s disease (glucogenosis type II) is caused by acid maltase deficiency and is discussed in question 48.

Ropper AH, Samuels MA. Adams and Victor’s Principles of Neurology, 9th ed. New York: McGraw-Hill; 2009.

15. d, 16. a

The patient depicted in question 15 is presenting in myasthenic crisis. In patients with signs of respiratory muscle involvement, negative inspiratory force and forced vital capacity should be monitored closely. Intubation and mechanical ventilation should be initiated when the negative inspiratory force is less then −20 cm of H2O, forced vital capacity less than 15 mL/kg, or if there is a significant downward trend of spirometry measures with clinical evidence of respiratory muscle fatiguing.

The patient should be admitted to the ICU and intubated and mechanically ventilated. In patients with significant oculobulbar symptoms, neck flexor weakness (suggesting potential involvement of respiratory muscles), or respiratory muscle weakness, initial treatment is with either intravenous immunoglobulins or plasmapheresis.

Medications that exacerbate myasthenia gravis include aminoglycosides, β-blockers, and neuromuscular-blocking agents, among others. These should be avoided or used at the lowest possible dose in myasthenia gravis; symptom exacerbation with initiation of any medication should prompt investigation as to whether or not the medication is the culprit. Penicillamine can cause a seropositive myasthenic syndrome.

The majority of patients with myasthenia gravis, except some with purely ocular myasthenia, require immunosuppressive therapy. Patients are often treated with corticosteroids for several weeks to months until secondary immunosuppressive therapy has taken effect. Approximately one-third of patients will have worsening of their myasthenic symptoms at the onset of steroid therapy that lasts up to 10 days. Therefore, caution is required with the initiation of steroid therapy. When symptoms are severe, and there is concern for significant pharyngeal or respiratory muscle involvement, initiation of plasma exchange or intravenous immunoglobulins prior to starting corticosteroids may be necessary. Secondary immunosuppressive agents that may be used include azathioprine, mycophenolate mofetil, cyclophosphamide, and cyclosporine. Pyridostigmine provides symptomatic relief but does not modify the course of the illness. The acetylcholine esterase inhibitors are discussed in question 22.

In the majority of patients with autoimmune myasthenia gravis, thymic abnormalities are present, most commonly lymphoid follicular hyperplasia. In a minority of patients, benign thymoma is present, and rarely, malignant thymoma. Resection of the thymus (regardless of whether or not thymoma is present) can potentially induce remission of myasthenia gravis. In autoimmune myasthenia gravis, there are ongoing clinical trials to determine the utility of thymectomy. However, there is evidence that patients who undergo thymectomy are more likely to attain remission of their myasthenia gravis. Thymectomy is therefore recommended for patients with myasthenia gravis with symptom onset prior to the age of 60, especially in younger women who seem to show the most benefit. Response to resection may not be apparent for several months to years. Because myasthenic symptoms can worsen in the perioperative period, stabilization of symptoms with preoperative intravenous immunoglobulins or plasma exchange prior to operation may be indicated.

Bradley WG, Daroff RB, Fenichel GM, et al. Neurology in Clinical Practice, 5th ed. Philadelphia, PA: Elsevier; 2008.

Gronseth GS, Barohn RJ. Practice parameters: Thymectomy for autoimmune myasthenia gravis (an evidence-based review). Report of the Quality Standards Subcommittee of the American Academy of Neurology. Neurology. 2000; 55:7–15.

Ropper AH, Samuels MA. Adams and Victor’s Principles of Neurology, 9th ed. New York: McGraw-Hill; 2009.

17. a

This patient has paramyotonia congenita, which is an autosomal dominant channelopathy caused by a mutation in the sodium channel gene SCN4A. The manifestations are very similar to those of myotonia congenita; however, there is no “warm-up” phenomenon and the symptoms are worsened with exercise and exposure to cold, features that help differentiate this condition from myotonia congenita. Percussion myotonia is rare, but can be seen after exposure to cold. EMG after cold exposure can also demonstrate fibrillation potentials followed by electrical inexcitability.

Becker’s and Thomsen’s disease are discussed in question 63. Hyperkalemia and hypo- kalemic periodic paralysis are discussed in questions 1 and 38.

Saperstein DS. Muscle channelopathies. Semin Neurol. 2008; 28:260–269.

18. a

This patient’s history is consistent with vasovagal syncope, a form of neurally mediated syncope or neurocardiogenic syncope. Syncope is a sudden transient loss of consciousness with a loss of postural tone. There are several causes of syncope; one of the most common causes is vasovagal syncope. It can have many potential triggers, with phlebotomy being a common one. This type of syncope results from a combination of inhibition of normal vascular sympathetic tone and increased vagal tone. Clinical features that suggest vasovagal syncope include female sex, and preceding or concomitant diaphoresis, palpitations, and nausea.

The other options are all potential causes of syncope. Glossopharyngeal neuralgia is characterized by severe pain in the hypopharynx and pharynx, tongue, and ear, with hypotension and bradycardia that when severe may be associated with syncope. It is often posttraumatic, but can occur with neck tumors as well. There are no features in the case to suggest glossopharyngeal neuralgia. A lack of postictal confusion and absence of convulsions helps distinguish syncope from seizures in this case. Carotid sinus hypersensitivity is discussed in question 13; syncope in the latter disorder is triggered by maneuvers that increase pressure on the carotid sinus, such as a tight collar. A third ventricular tumor, such as a colloid cyst, can lead to syncope with postural changes if the change in posture leads to obstruction of the third ventricle, leading to increased intracranial pressure. The history in this case is more suggestive of vasovagal syncope.

Bradley WG, Daroff RB, Fenichel GM, et al. Neurology in Clinical Practice, 5th ed. Philadelphia, PA: Elsevier; 2008.

19. b, 20. a

This patient has Duchenne muscular dystrophy. This condition is a dystrophinopathy, inherited in an X-linked recessive fashion, and is the most common genetic muscle disease, affecting 1 in 3500 live male births. About a third of the cases are caused by spontaneous mutations in the dystrophin gene, so there may not be a positive family history. This condition presents early in life and manifests with weakness and delayed development of motor milestones. These children have frequent falls, and difficulty walking, running, and rising from supine and sitting positions. Weakness is significant in the proximal muscles, predominantly in the iliopsoas, quadriceps, and gluteals, as well as the shoulder girdle and upper limbs. It also tends to affect the pretibial muscles. These patients have pseudohypertrophy of the calves due to fibrosis. They also have scapular winging and contractures. Ocular, facial, and bulbar muscles are usually spared. Cardiac involvement includes arrhythmias, cardiomyopathy, and heart failure. Mild mental retardation with a subnormal intelligence quotient (IQ) may be seen. These children become wheelchair- or bed-bound, and may eventually die of respiratory failure and pulmonary infections. Creatine kinase values range from 10 to 100 times higher than normal. Needle EMG shows fibrillations, positive waves, and myopathic motor unit potentials.

The muscle biopsy findings in Figure 10.1 indicate a muscular dystrophy, showing endomysial fibrosis, loss of muscle fibers with residual fibers of different sizes, some of which are very large and eosinophilic, and others very small and atrophic. Degeneration and regeneration, as well as necrosis and macrophage invasion, may also be seen. In general, myofiber necrosis, degeneration, regeneration, increased fiber size variation, endomysial inflammation, and fibrosis are collectively called dystrophic changes, which are characteristic for Duchenne muscular dystrophy, but not specific, and can be seen in other dystrophies.

Dystrophin is the affected gene product and is absent in patients with Duchenne. Dystrophin is a cytoplasmic protein that binds and interacts with other intracytoplasmic proteins such as F-actin, and is linked to the trans-sarcolemmal dystroglycan protein complex, providing a structural link between the subsarcolemmal cytoskeleton and the extracellular matrix. If dystrophin is lost or altered, this sarcolemmal structure is affected, making it susceptible to rupture. The diagnosis of Duchenne muscular dystrophy is made by genetic testing of the dystrophin gene or by absent dystrophin immunostaining on muscle biopsy.

Becker muscular dystrophy is similar to Duchenne muscular dystrophy; however, it is much less severe, with a later onset (usually in adolescence or adulthood), which is not the case in this patient. Becker muscular dystrophy is discussed in questions 30 and 31. The other options are unlikely to be correct diagnosis in this case. Emery-Dreifuss muscular dystrophy is discussed in questions 51 and 52. Fascioscapulohumeral muscular dystrophy is discussed in question 9. Limb-girdle muscular dystrophy is discussed in questions 41 and 42.

Prayson RA, Goldblum JR. Neuropathology, 1st ed. Philadelphia, PA: Elsevier; 2005.

Ropper AH, Samuels MA. Adams and Victor’s Principles of Neurology, 9th ed. New York: McGraw-Hill; 2009.

21. b

This patient has Nonaka myopathy, which is an autosomal recessive distal myopathy, with onset in early adulthood. It is characterized by foot drop associated with weakness of the anterior tibial muscles. Eventually, muscles of the upper extremities are affected, especially the extensors. Rarely, there is proximal weakness and no bulbar involvement. Histologically, the muscle biopsy shows rimmed vacuoles, and electron microscopy shows tubular filaments similar to those seen in inclusion body myositis, without inflammation. This condition is associated with a mutation in the GNE gene located on chromosome 9p.

Welander muscular dystrophy is a distal myopathy inherited in an autosomal dominant fashion, with an onset later in adult life usually between 40 and 60 years of age (not consistent with this case). Welander myopathy usually begins with weakness and atrophy in the distal muscles of the hands and later affects the legs. Muscle biopsy demonstrates myopathic changes and rimmed vacuoles.

Markesbery-Griggs is also an autosomal dominant distal myopathy with onset later in adult life, caused by mutations in the gene encoding titin on chromosome 2q. It is characterized by foot drop and the later development of wrist drop and weakness of extensor muscles of the forearms. The biopsy findings are similar to those seen in Welander myo- pathy.

Miyoshi myopathy and scapuloperoneal muscular dystrophy are not consistent with this case and are discussed in question 26.

Cardamone M, Darras BT, Ryan MM. Inherited myopathies and muscular dystrophies. Semin Neurol. 2008; 28:250–259.

Ropper AH, Samuels MA. Adams and Victor’s Principles of Neurology, 9th ed. New York: McGraw-Hill; 2009.

22. c

Acetylcholine esterase inhibitors, including pyridostigmine and neostigmine, inhibit acetylcholine esterase, increasing levels of acetylcholine at the neuromuscular junction (NMJ). The most commonly used acetylcholine esterase inhibitor in myasthenia gravis is pyridostigmine. It improves symptoms, but does not alter the course of the disease. Side effects are due to activation of muscarinic receptors outside of the NMJ, and include diarrhea, nausea, abdominal cramps, and increased bronchial secretions. At therapeutic doses, pyridostigmine does not affect central cholinergic pathways, as it does not cross the blood-brain barrier.

With excessive dosing, cholinergic crisis can occur, with paradoxical worsening of weakness, even leading to respiratory failure. Patients with myasthenia presenting with significant worsening of symptoms with respiratory involvement should be assessed for the possibility of cholinergic crisis rather than myasthenic crisis. Cholinergic crisis results from overstimulation of muscarinic receptors not only at the NMJ but also systemically due to overdosing with pyridostigmine. Symptoms that distinguish cholinergic crisis from myasthenic crisis include the presence of nausea, vomiting, diaphoresis, sialorrhea, excessive bronchial secretions, miosis, bradycardia, and diarrhea (see also Chapter 3).

Bradley WG, Daroff RB, Fenichel GM, et al. Neurology in Clinical Practice, 5th ed. Philadelphia, PA: Elsevier; 2008.

Ropper AH, Samuels MA. Adams and Victor’s Principles of Neurology, 9th ed. New York: McGraw-Hill; 2009.

23. d

The muscle biopsy shown in Figure 10.2 is consistent with a diagnosis of central core myopathy. This nicotinamide adenine dinucleotide-stained specimen demonstrates loss of oxidative activity within the center of muscle fibers, seen as paler areas where there is absence of mitochondria. These central cores run along the length of the muscle fiber, as opposed to multicore or minicore disease in which the cores are small, extending through only segments of the fiber length.

Central core myopathy is an autosomal dominant disease, caused by a mutation in the ryanodine receptor gene RYR1 on chromosome 19q13.1. These patients are at risk for the development of malignant hyperthermia and this should be considered when general anesthesia is needed. The clinical presentation is that of weakness and hypotonia soon after birth, and subsequent delay in motor development. The weakness is proximal, and the pelvic girdle is usually more affected than the shoulder girdle. Facial, bulbar, and ocular muscles are usually spared. Creatine kinase levels are slightly elevated.

Cardamone M, Darras BT, Ryan MM. Inherited myopathies and muscular dystrophies. Semin Neurol. 2008; 28:250–259.

Prayson RA, Goldblum JR. Neuropathology, 1st ed. Philadelphia, PA: Elsevier; 2005.

Ropper AH, Samuels MA. Adams and Victor’s Principles of Neurology, 9th ed. New York: McGraw-Hill; 2009.

24. b, 25. b

This patient’s history and examination are consistent with an autonomic ganglionopathy. Symptoms develop over weeks, as opposed to pure autonomic failure, in which symptoms develop slowly over time. Symptoms of autonomic ganglionopathy are due to dysfunction in the parasympathetic and sympathetic nervous system, and include orthostatic hypotension, absent heart rate variability (see question 47 on tilt table testing), hypohidrosis or anhidrosis, dry mouth and eyes, pupillary abnormalities, sexual dysfunction, early satiety, constipation, and diarrhea due to abnormal gastric and intestinal motility. It results from impairment of transmission of impulses in the autonomic ganglia and affects both the parasympathetic and sympathetic nervous systems. In approximately half of patients, an antibody against the ganglionic nicotinic acetylcholine receptor can be identified in the serum. Autonomic ganglionopathy can be either autoimmune or paraneoplastic, with small cell lung carcinoma being a common associated malignancy; these cannot definitively be distinguished between clinically, and the antibody can be present in either form. In patients with the autoimmune form, other autoimmune disorders may be present, and symptoms may be preceded by antecedent viral illness.

Treatment of autoimmune autonomic ganglionopathy includes plasma exchange and intravenous immunoglobulins; typically, some improvement of symptoms occurs, but full recovery is unlikely.

Acetylcholine receptor binding antibodies are seen in autoimmune myasthenia gravis. Antistriational muscle antibodies can be detected in patients with autoimmune myasthenia gravis or thymoma. Antibodies against presynaptic P/Q-type voltage-gated calcium channel are seen in Lambert-Eaton syndrome. Anti-Jo 1 antibodies are seen in polymyositis.

In a patient without a known history of diabetes, autonomic neuropathy would unlikely be the sole manifestation of nervous system involvement from diabetes. Postural orthostatic tachycardia syndrome is described in question 37, amyloid autonomic neuropathy in Chapter 9, and pure autonomic failure in question 34.

Bradley WG, Daroff RB, Fenichel GM, et al. Neurology in Clinical Practice, 5th ed. Philadelphia, PA: Elsevier; 2008.

26. a

This patient has Miyoshi myopathy, which is an autosomal recessive condition presenting early in adult life and manifesting with weakness and atrophy in the distal leg muscles, predominantly in the posterior compartment. The weakness may eventually affect more proximal muscle groups. This condition is caused by a mutation in the gene encoding dysferlin on chromosome 2p. Dysferlin is also associated with limb girdle muscular dystrophy type 2B. Creatine kinase levels are markedly elevated in the range of 10 to 100 times that of normal. Biopsy shows dystrophic changes.

Scapuloperoneal muscular dystrophy is a disorder presenting with muscular weakness and wasting, affecting the muscles of the neck, shoulders, upper arms, and anterior leg compartment causing foot drop. It may be inherited in an autosomal dominant fashion, but can also be X-linked. It is not the case in this patient.

Nonaka, Markesbery-Griggs, and Welander myopathy are not consistent with this case, and are discussed in question 21.

Cardamone M, Darras BT, Ryan MM. Inherited myopathies and muscular dystrophies. Semin Neurol. 2008; 28:250–259.

Ropper AH, Samuels MA. Adams and Victor’s Principles of Neurology, 9th ed. New York: McGraw-Hill; 2009.

27. e

This patient’s history and examination are consistent with myasthenia gravis with anti-muscle specific tyrosine kinase (MuSK) antibodies. In approximately half of all patients who are seronegative for antibodies against the acetylcholine receptor, anti-MuSK antibodies are present. In patients with anti-MuSK antibodies, the clinical picture may resemble the typical form of autoimmune myasthenia gravis or ocular myasthenia, but more commonly, weakness involves predominantly cranial and bulbar muscles, with prominent dysphagia, neck flexor weakness, and respiratory weakness, with relative sparing of ocular muscles (though eyelid and ocular muscle involvement may occur). This disorder is more common in young women, and typically does not respond to pyridostigmine. Treatment is with immune-modulating therapy, including intravenous immunoglobulins and plasmapheresis.

Anti-striational muscle antibodies would not be of utility in this case; they are discussed in question 6. Antibodies against the presynaptic calcium channel are seen in Lambert-Eaton syndrome (discussed in questions 43 and 44); features of this syndrome including hyporeflexia, transient improvement of symptoms with sustained contraction, and autonomic features are not present in this case. Anti-Jo 1 antibodies are seen in polymyositis; serum creatine kinase is often elevated, and EMG shows evidence of myopathy (discussed in question 35). Anti-GQ1b antibodies are seen in Miller-Fisher syndrome (discussed in Chapter 9).

Bradley WG, Daroff RB, Fenichel GM, et al. Neurology in Clinical Practice, 5th ed. Philadelphia, PA: Elsevier; 2008.

28. d

Acetylcholine acts at muscarinic M2-receptors in the heart to reduce heart rate. The main neurotransmitters of the autonomic nervous system include norepinephrine and acetylcholine, although several neuropeptide neurotransmitters colocalize with these neurotransmitters and exert actions that affect autonomic nervous system function as well. Epinephrine is released by the adrenal medulla, and exerts similar actions to norepinephrine at α-receptors. See Table 10.1 for some of the effects of acetylcholine and norepinephrine at various receptors.

TABLE 10.1 Actions of norepinephrine and acetylcholine at various receptors

Brunton LL. Goodman and Gilman’s the Pharmacological Basis of Therapeutics, 11th ed. New York: McGraw-Hill; 2006.

Ropper AH, Samuels MA. Adams and Victor’s Principles of Neurology, 9th ed. New York: McGraw-Hill; 2009.

29. c

The congenital muscular dystrophies are a group of disorders mostly inherited in an autosomal recessive fashion. In utero, there may be decreased movements. At birth, these children are hypotonic and weak, presenting with arthrogryposis, and may develop respiratory insufficiency and bulbar dysfunction. With time they may develop contractures and scoliosis. These patients also have mental retardation and developmental anomalies of the cerebral cortex. Creatine kinase is markedly elevated. EMG shows myopathic changes. Histopathologically, there are dystrophic changes with degeneration and regeneration of muscle fibers, and infiltration of connective tissue. The congenital muscular dystrophies are caused by genetic defects in sarcolemmal membrane proteins or membrane-supporting structures.

Cardamone M, Darras BT, Ryan MM. Inherited myopathies and muscular dystrophies. Semin Neurol. 2008; 28:250–259.

Ropper AH, Samuels MA. Adams and Victor’s Principles of Neurology, 9th ed. New York: McGraw-Hill; 2009.

30. a, 31. c

This patient has Becker muscular dystrophy, which is a milder dystrophinopathy similar to Duchenne muscular dystrophy, but in which the dystrophin protein is not completely absent, but may be structurally abnormal or present in a smaller amount.

Becker muscular dystrophy is an X-linked recessive disorder, which presents with weakness and involvement of the same muscle groups as in Duchenne muscular dystrophy, as well as with pseudohypertrophy of the calves. However, patients with Becker muscular dystrophy present later in life, sometimes in childhood, but more frequently in adolescence or adulthood. Given the milder phenotype, these patients are able to ambulate beyond the second decade of life, cardiac involvement occurs but is less frequent, and intelligence quotient (IQ) is usually normal. Needle EMG shows fibrillations, positive waves, and polyphasic motor unit potentials. The biopsy will detect similar findings as in Duchenne muscular dystrophy (see questions 19 and 20); however, dystrophin is not absent in Becker muscular dystrophy, but structurally abnormal or present in less amount.

The other options are not likely to be the diagnosis in this case. Duchenne muscular dystrophy is discussed in questions 19 and 20. Emery-Dreifuss muscular dystrophy is discussed in questions 51 and 52. Fascioscapulohumeral muscular dystrophy is discussed in question 9. The myotonic dystrophies are discussed in questions 7 and 12.

Prayson RA, Goldblum JR. Neuropathology, 1st ed. Philadelphia, PA: Elsevier; 2005.

Ropper AH, Samuels MA. Adams and Victor’s Principles of Neurology, 9th ed. New York: McGraw-Hill; 2009.

32. a, 33. a

This patient has dermatomyositis. The use of methotrexate should be avoided if patients have interstitial lung disease because this medication can be toxic to the lung.

Dermatomyositis is an inflammatory condition of the muscle associated with skin findings. The muscle weakness is a typical myopathic weakness, predominantly proximal. Therefore, the patient will complain of being unable to stand from the sitting position, inability to walk up stairs, and difficulty lifting the arms above the head. Typical skin findings are the heliotrope rash (purplish discoloration of the eyelids), erythema of the face, neck, anterior chest, upper back, elbows and knees, Gottron’s papules (purplish scaly papular rash on the extensor surface of the hands), and “mechanic’s hands” (thickened skin on the dorsal and ventral surface of the hands. Serum creatine kinase levels may be normal or elevated. The muscle biopsy specimen shown in Figure 10.3 is characteristic for dermatomyositis, showing atrophic fibers in the borders of the fascicles, which is known as perifascicular atrophy.

Dermatomyositis is associated with malignancy and therefore these patients should have age-appropriate malignancy screening. Dermatomyositis is also associated with interstitial lung disease, especially if anti-Jo antibody is positive; this antibody is a marker of this association.

The initial treatment of choice is steroids; however, long-term steroid-sparing agents may be required, such as azathioprine or methotrexate. As mentioned above, methotrexate may be associated with pulmonary toxicity, and therefore if there is interstitial lung disease, this medication should be avoided.

On the basis of clinical and pathologic findings, this patient does not have polymyositis, inclusion body myositis, mitochondrial myopathy, or muscular dystrophy.

Greenberg SA. Inflammatory myopathies: Evaluation and management. Semin Neurol. 2008; 28:241–249.

Prayson RA, Goldblum JR. Neuropathology, 1st ed. Philadelphia, PA: Elsevier; 2005.

34. e

This patient’s history is consistent with pure autonomic failure, also known as Bradbury-Eggleston syndrome, which has been attributed to loss of intermediolateral cell column neurons. It results from deposition of α-synuclein in the autonomic nervous system. Pathologically, Lewy bodies are seen (see Chapter 12). It presents in mid to late adulthood. In men, the earliest symptom is typically impotence. Orthostatic hypotension (defined as a reduction of systolic blood pressure by 20 mm Hg or diastolic blood pressure by 10 mm Hg), or a pulse rate increase of 20 bpm, is present, and is often worse in the morning, after meals, exertion, and with heat exposure. Supine hypertension may occur as well. Other features include hypohidrosis, nocturia, early satiety and nausea (due to gastrointestinal hypomotility), urinary hesitance and/or urgency with occasional incontinence, and neck and shoulder aching (“coat hanger” distribution) precipitated by standing.

In multiple system atrophy (discussed in Chapter 6), dysautonomia is typically more severe and disabling, and extrapyramidal or cerebellar findings are present on examination. Autoimmune autonomic ganglionopathy is discussed in questions 24 and 25. Familial amyloid polyneuropathy type I can lead to autonomic neuropathy (see Chapter 9), but the history and examination (specifically, absence of dissociated sensory loss on examination) make this less likely. This patient is not known to have diabetes, and a normal sensory examination further makes diabetic autonomic neuropathy unlikely.

Bradley WG, Daroff RB, Fenichel GM, et al. Neurology in Clinical Practice, 5th ed. Philadelphia, PA: Elsevier; 2008.

Ropper AH, Samuels MA. Adams and Victor’s Principles of Neurology, 9th ed. New York: McGraw-Hill; 2009.

35. d

The muscle biopsy in Figure 10.4 demonstrates an inflammatory infiltrate composed of lymphocytes and macrophages surrounding muscle fibers. There are regenerating and necrotic fibers. These findings are characteristic of polymyositis, and neither rimmed vacuoles nor perifascicular atrophy are seen. Rimmed vacuoles are seen in inclusion body myositis. Perifascicular atrophy is seen in dermatomyositis.

Polymyositis is an inflammatory condition of the muscle in which there are no skin findings. The weakness is proximal, and this condition, similar to dermatomyositis, has been associated with interstitial lung disease, and anti-Jo antibody may be a marker of this association. On the other hand, polymyositis as opposed to dermatomyositis does not have a strong correlation with the presence of malignancy. The initial treatment of choice is steroids; however, in the long term, a steroid-sparing agent such as methotrexate or azathioprine may be required.

Greenberg SA. Inflammatory myopathies: Evaluation and management. Semin Neurol. 2008; 28:241–249.

Prayson RA, Goldblum JR. Neuropathology, 1st ed. Philadelphia, PA: Elsevier; 2005.

36. e

Concomitant use of statins and fibrates increases the risk of muscle toxicity.

Statins are lipid-lowering agents that inhibit the 3-hydroxy-3-methyl-glutaryl-coenzyme A reductase. Statins are known to have muscle toxic effects, and the possible mechanism of this adverse effect is by an action on the mitochondria and sarcoplasmic reticulum, especially in type II muscle fibers. A mutation in the gene SLCO1B has been associated with predisposition to develop statin-induced myopathy.

Various types of toxic muscle effects are seen clinically:

– Increased serum creatine kinase (CK) in asymptomatic patients, with mild elevations and rarely up to 10 times the upper limit of normal.
– Myalgia with or without increased serum CK levels. Usually, muscle strength is normal, and myalgias may improve after discontinuation of the medication.
– Muscle weakness with serum CK elevation.
– Rhabdomyolysis. In this condition, the CK levels are markedly elevated, sometimes above 15,000 IU/L (normal up to 220 IU/L). These CK elevations could lead to myoglobinuria and renal failure, such as in this case.

Concomitant use of fibrates can increase the risk of developing muscle toxicity. This risk can also be increased by the use of other drugs, such as azole antifungals, macrolides antibiotics, antiretrovirals, and amiodarone, among others. Therefore a careful assessment of the medications that the patient is taking is important when treating a patient with statins.

Dalakas MC. Toxic and drug-induced myopathies. J Neurol Neurosurg Psychiatry. 2009; 80:832–838.

37. d

This patient’s history and tilt table test results are consistent with postural orthostatic tachycardia syndrome (POTS), the most common form of dysautonomia.

Tilt table examination is used to assess changes in heart rate and blood pressure that occur with changes in posture, increased vagal tone (as occurs with valsalva), and sometimes, in response to the administration of pharmacologic agents such as the β-agonist isoproterenol. In POTS, tilt table examination shows an increase in heart rate of at least 30 bpm from baseline, or up to more than 120 bpm within 10 minutes of head-up tilt, without significant changes in blood pressure, but with symptoms of orthostasis such as light-headedness, palpitations, generalized weakness, visual changes, headache, or tremor. The etiology of POTS is unclear; it is more common in females, and symptoms worsen around menses, suggesting a role for estrogen in the pathophysiology. Symptom onset may follow a variety of triggers, including viral infection, pregnancy, or surgery, and for this reason, autoimmune causes have also been postulated. POTS can rarely be a manifestation of a mutation in a norepinephrine transporter gene, which leads to elevated plasma levels of norepinephrine, with subsequent sympathetic overactivity. POTS may be comorbid with chronic fatigue syndrome; the significance of this relationship is not clear. In some patients with POTS, the symptoms are mild and do not require treatment. In others, treatment may include β-blockers, increased fluid and salt intake, fludrocortisone, and midodrine.

There is not a reduction in blood pressure in POTS, distinguishing it from orthostatic hypotension. With vasovagal response, there is no tachycardia but rather transient bradycardia due to parasympathetic overactivation. Systemic mastocytosis, a hematologic disorder in which mast cells are overactive, leading to excessive histamine release, can lead to POTS-like symptoms; skin changes such as flushing and an absence of clear postural component distinguish the two. Subclavian steal syndrome results in syncope in the setting of stenosis of the subclavian artery proximal to the origin of the left vertebral artery. With exercise of the left arm, blood flow steal from the vertebral arteries and basilar artery may occur, resulting in syncope and other symptoms of basilar insufficiency. There is no indication in the case that the syncope followed left arm exercise.

Bradley WG, Daroff RB, Fenichel GM, et al. Neurology in Clinical Practice, 5th ed. Philadelphia, PA: Elsevier; 2008.

Ropper AH, Samuels MA. Adams and Victor’s Principles of Neurology, 9th ed. New York: McGraw-Hill; 2009.

38. d

This patient has manifestations suggesting a type of periodic paralysis. Adult-onset periodic paralysis suggests a possible secondary cause, of which thyrotoxic periodic paralysis is the best recognized. Therefore, thyroid-stimulating hormone (TSH) should be obtained, especially given the history of palpitations and possibly anxiety.

Thyrotoxic periodic paralysis is a condition associated with hyperthyroidism, thought to be present in genetically susceptible patients but without a known gene mutation detected to date. It is more common in Asians, with a male predominance. These patients have episodes of proximal weakness and other manifestations of thyrotoxicosis. During acute attacks, potassium should be provided. β-blockers may be of benefit as prophylaxis. Carbonic anhydrase inhibitors do not work.

In this case, the best next step is checking a TSH level. Needle EMG is not specific. Genetic testing is expensive and should only be done when a specific mutation is suspected, and in this case, a thyroid problem should be ruled out first. Referral to psychiatry is not indicated in this case, and as mentioned above, carbonic anhydrase inhibitors do not play a role in thyrotoxic periodic paralysis.

Saperstein DS. Muscle channelopathies. Semin Neurol. 2008; 28:260–269.

39. c, 40. a

This patient has inclusion body myositis, which is a myopathy that affects adults usually older than 50 years. It is characterized by asymmetric weakness and atrophy of the wrist and finger flexors, quadriceps, and anterior tibial muscles. Dysphagia can be present, and there are no skin manifestations. Creatine kinase may only be slightly elevated.

This biopsy specimen shown in Figure 10.5 demonstrates endomysial inflammation, groups of atrophic fibers, and intracytoplasmatic vacuoles with granular material known as rimmed vacuoles, which are characteristic of inclusion body myositis.

This condition does not respond to steroids, and effective therapies are currently not available.

Perifascicular atrophy, Gottron’s papules, and “mechanic hands” are seen in dermatomyositis (discussed in questions 32 and 33), but not in inclusion body myositis.

Greenberg SA. Inflammatory myopathies: Evaluation and management. Semin Neurol. 2008; 28:241–249.

Prayson RA, Goldblum JR. Neuropathology, 1st ed. Philadelphia, PA: Elsevier; 2005.

41. a

Cardiac involvement is evident in various types of limb girdle muscular dystrophies (LGMD).

LMGD are a group of muscular dystrophies in which there is proximal weakness with involvement of the shoulder or pelvic girdle with sparing of the facial muscles. It can present in both sexes, and the inheritance is either autosomal dominant or recessive. The mode of inheritance determines the current classification, which has mainly been based on the genetic defects and gene products involved. Regarding classification by inheritance, LGMD1 is autosomal dominant and LGMD 2 is autosomal recessive. Needle EMG shows myopathic changes. Muscle biopsy shows dystrophic changes, with myofiber necrosis, degeneration, regeneration, increased fiber size variation, endomysial inflammation, and fibrosis.

Various types of LGMDs may be associated with cardiomyopathy, and some other types are associated with joint contractures.

Cardamone M, Darras BT, Ryan MM. Inherited myopathies and muscular dystrophies. Semin Neurol. 2008; 28:250–259.

Ropper AH, Samuels MA. Adams and Victor’s Principles of Neurology, 9th ed. New York: McGraw-Hill; 2009.

42. d

As mentioned in the discussion to question 41, the limb girdle muscular dystrophies (LGMDs) are a large group of disorders in which patients present with proximal weakness. There is large variability in terms of presentation, and classification of the LGMDs has changed over time. Most recently, they have been classified on the basis of the gene product affected.

There are several types of LGMDs associated with various gene products (Tables 10.2 and 10.3).

TABLE 10.2 Autosomal dominant limb girdle muscular dystrophies (LGMDs)

TABLE 10.3 Autosomal recessive limb girdle muscular dystrophies (LGMDs)

Cardamone M, Darras BT, Ryan MM. Inherited myopathies and muscular dystrophies. Semin Neurol. 2008; 28:250–259.

Ropper AH, Samuels MA. Adams and Victor’s Principles of Neurology, 9th ed. New York: McGraw-Hill; 2009.

43. b, 44. e

The history and examination of the patient depicted in question 43 are consistent with Lambert-Eaton myasthenic syndrome (LEMS). LEMS is a form of myasthenia. It is a neuromuscular junction (NMJ) disorder resulting from antibodies against presynaptic P/Q-type voltage-gated calcium channel, reducing the influx of calcium that normally leads to release of acetylcholine into the NMJ. It is more common in males. As compared to autoimmune myasthenia gravis, it is less likely to involve ocular or bulbar musculature; it predominantly affects proximal (shoulder and hip girdle) and trunk musculature. Proximal leg weakness is a common clinical presentation of LEMS, and the weakness perceived by the patient may be out of proportion to the examination. While in LEMS, worsening of muscle weakness with repetitive muscle use ultimately occurs, there is a transient improvement of muscle strength following repetitive muscle use prior to worsening weakness. This can sometimes be elicited on physical examination as well as electrophysiologically. Also, unlike myasthenia gravis, in LEMS, deep tendon reflexes are often diminished or absent. Autonomic features are prominent in LEMS, including constipation, urinary retention, and impotence.

LEMS may be either autoimmune or paraneoplastic; paraneoplastic LEMS is most commonly associated with small cell lung cancer. A diagnosis of autoimmune LEMS should be made only after excluding an occult malignancy with an extensive evaluation and after long-term surveillance for malignancy. The SOX antibody has been identified as being a useful marker to distinguish between squamous cell lung cancer–associated paraneoplastic LEMS (in which SOX antibody is present) and autoimmune LEMS.

Unlike myasthenia gravis, patients with LEMS respond little if at all to acetylcholine esterase inhibitors. Symptomatic treatment of LEMS may be achieved with 3,4-diaminopyridine (3,4-DAP), a drug which inhibits presynaptic potassium channels, prolonging depolarization and increasing acetylcholine vesicle exocytosis. Autoimmune LEMS is treated with immunosuppressive therapy, including corticosteroids as well as steroid-sparing agents, intravenous immunoglobulins, and plasmapheresis. Response to these treatments is variable and is typically not as successful as in myasthenia gravis. In paraneoplastic LEMS, treatment of the malignancy may improve the neuromuscular symptoms.

EMG shows low-amplitude CMAPs that increase in amplitude following exercise (as in Figure 10.6, in which there is a >50% increase in CMAP amplitude postexercise). This is a phenomenon known as facilitation. On high-frequency repetitive nerve stimulation, a prominent increment is seen, in contrast to the electrodecrement seen with low-frequency repetitive nerve stimulation in myasthenia gravis (discussed in question 6 and Chapter 9).

The history, examination, normal creatine kinase, and EMG/NCS findings make a myopathy or myositis unlikely. Similarly, a demyelinating polyneuropathy would have been detected on NCS, and facilitation of reflexes with both myopathies and demyelinating polyneuropathy does not occur.

Bradley WG, Daroff RB, Fenichel GM, et al. Neurology in Clinical Practice, 5th ed. Philadelphia, PA: Elsevier; 2008.

Sabater L, Titulaer M, Saiz A, et al. SOX1 antibodies are markers of paraneoplastic Lambert-Eaton myasthenic syndrome. Neurology. 2008; 70:924–928.

45. d

This patient has trichinosis. The patient was previously healthy, now with an acute to subacute onset of symptoms, mildly elevated creatine kinase (CK) and markedly elevated eosinophils, which make the other diagnoses less likely. Furthermore, the muscle biopsy demonstrates the presence of a parasite, confirming the diagnosis of trichinosis.

Trichinosis is caused by the intestinal nematode Trichinella spiralis, which is transmitted by the ingestion of uncooked pork containing the encysted larvae. Once ingested, the larvae are liberated from the cysts in the gastrointestinal tract, developing into adult worms in the duodenum or jejunum. The female worms deposit batches of larvae in the intestinal wall, which migrate via lymphatics into the bloodstream. These larvae invade all tissues, but survive only in muscles, where they become encysted and calcify.

Patients develop a mild gastroenteritis on ingestion of infected meat. About 1 week later and sometimes for up to 4 to 6 weeks, patients may experience low-grade fever, myalgias, fatigue, and edema of the conjunctiva and eyelids. Ocular muscle weakness occurs, resulting in strabismus and diplopia. Also, weakness of the tongue and masticatory as well as pharyngeal muscles occurs, which may cause dysarthria and dysphagia. Limb muscle weakness occurs, more prominent proximally. The diaphragm and cardiac muscles may also be involved. Myopathic symptoms eventually subside and patients generally improve completely. However, massive infections have been reported to cause CNS symptoms, including headaches, neck stiffness, and confusion.

Laboratory studies demonstrate eosinophilia and moderate elevations of CK. Enzyme-linked immunosorbent assay blood test for trichinella becomes positive after 1 to 2 weeks and may be helpful. Muscle biopsy, such as in this case, as shown in Figure 10.7, will demonstrate the parasite and sometimes an inflammatory infiltrate with eosinophils.

Treatment includes a combination of thiabendazole with steroids. Thiabendazole is an anti-helminthic that prevents larval reproduction and interferes with the metabolism of the larvae in the muscle. Albendazole is also effective.

Bradley WG, Daroff RB, Fenichel GM, et al. Neurology in Clinical Practice, 5th ed. Philadelphia, PA: Elsevier; 2008.

Ropper AH, Samuels MA. Adams and Victor’s Principles of Neurology, 9th ed. New York: McGraw-Hill; 2009.

46. c

Nemaline myopathy has phenotypic variability, with presentations ranging from severe neonatal congenital forms to adult-onset forms in which patients have proximal weakness, cardiomyopathy, and prominent compromise of respiratory muscles. Neonatal forms present with dysmorphic features, contractures, arthrogryposis, generalized hypotonia, feeding difficulties, and respiratory problems. There are other intermediate forms with infantile onset, childhood- and adolescent-onset forms.

This condition is inherited in an autosomal dominant or recessive fashion, and the genes implicated include α-actin, α-tropomyosin, and β-tropomyosin, nebulin, troponin, and cofilin. Histopathologically, the fibers have rod-like structures that are seen beneath the sarcolemma, also known as nemaline bodies or nemaline rods. Type I fibers are smaller than normal and predominate.

Cardamone M, Darras BT, Ryan MM. Inherited myopathies and muscular dystrophies. Semin Neurol. 2008; 28:250–259.

Ropper AH, Samuels MA. Adams and Victor’s Principles of Neurology, 9th ed. New York: McGraw-Hill; 2009.

47. b

Normally, with assumption of an upright posture, there is a transient reduction in blood pressure that is soon counteracted by an increase in heart rate mediated by inhibition of the parasympathetic nervous system and activation of the sympathetic nervous system. Splanchnic, renal, and skeletal muscle blood vessels vasoconstrict. The increase in heart rate is maximal at the 15th beat and a mild reflex bradycardia is noted at the 30th beat. In moderate or severe autonomic neuropathy, the compensatory increase in heart rate does not occur.

Upright tilt table evaluation is used to obtain objective measures of blood pressure and heart rate in various postures. In normal individuals, during the tilt table test, at 60 to 80 degrees of tilting for 10 minutes, there is a brief 5 to 15 mm Hg reduction in systolic blood pressure, a 5 to 10 mm Hg increase in diastolic blood pressure, and a 10 to 15 bpm increase in heart rate. If there is early, gradual, and sustained hypotension (>20 mm Hg reduction in systolic blood pressure) without compensatory tachycardia, this is an indication of insufficient sympathetic tone and impaired baroreceptor function. Hypotension with compensatory tachycardia suggests hypovolemia. If hypotension is delayed for several minutes but then occurs abruptly with bradycardia, this is more suggestive of a neurocardiogenic mechanism. Most neurogenic causes of syncope lead to gradual hypotension without a compensatory tachycardia that occurs within the first 5 to 10 minutes of tilting.

The thermoregulatory sweat test is a test of sudomotor function; it is a qualitative test of sweating in which an indicator powder is applied to the subject who is then exposed to elevated temperatures. Sweat interacts with the powder, indicating areas of normal sweating and identifying areas of absent or reduced sweating. Different patterns of anhidrosis can be identified that correspond to different autonomic disorders. In large and small fiber neuropathies, distal anhidrosis occurs. In patients with multiple system atrophy (see Chapter 6), pure autonomic failure (discussed in question 34), and autonomic neuropathies, global hypohidrosis or anhidrosis occurs.

The quantitative sudomotor axon reflex test is another test of sudomotor function. A sweat cell is applied to the skin and an electric current is then applied. Acetylcholine is iontophoresed into the skin, stimulating sweat glands. Sweat production is measured from adjacent sweat glands that are stimulated via an axon reflex. If there is a low or absent response following application of the electric current, this indicates a lesion of the postganglionic sympathetic axon.

Bolis CL, Licinio J, Govoni S. Handbook of the Autonomic Nervous System in Health and Disease, New York: Marcel Dekker; 2003.

Bradley WG, Daroff RB, Fenichel GM, et al. Neurology in Clinical Practice, 5th ed. Philadelphia, PA: Elsevier; 2008.

Ropper AH, Samuels MA. Adams and Victor’s Principles of Neurology, 9th ed. New York: McGraw-Hill; 2009.

48. c, 49. b

These patients have glycogenosis type II or acid maltase deficiency.

Glycogenosis type II or acid maltase deficiency is an autosomal recessive condition caused by deficiency of acid maltase (also known as α-1,4-glucosidase) in lysosomes. This enzyme participates in the breakdown of glycogen to glucose; its deficiency leads to glycogen accumulation, causing the typical histopathologic findings on muscle biopsy, demonstrating vacuolated sarcoplasm with glycogen accumulation that stains strongly with acid phosphatase.

There are three forms of acid maltase deficiency:

1. Pompe’s disease is the infantile form that presents in the first few months of life with difficulty feeding, cyanosis, dyspnea, macroglossia, hepatomegaly, cardiomegaly, and hypotonic weakness. This form progresses rapidly and patients die in the first few months.

2. Childhood form, which has an onset in the second year of life, manifests with proximal weakness, motor developmental delay, hypotonia, enlarged calves, and rarely with cardiomegaly, hepatomegaly, and mental retardation. These patients may die of pulmonary infections and respiratory failure.

3. Adult form, which presents in the second to fourth decades of life, manifests with slowly progressive proximal weakness and typically weakness of the diaphragm, leading to neuromuscular respiratory problems. These patients do not typically have cardiomegaly, hepatomegaly, or mental retardation.

Glycogenosis type V is McArdle’s disease. Glycogenosis type III is Cori’s disease. Both are discussed in question 14.

Ropper AH, Samuels MA. Adams and Victor’s Principles of Neurology, 9th ed. New York: McGraw-Hill; 2009.

50. c

This patient’s history and examination are consistent with congenital myasthenia due to congenital acetylcholine receptor deficiency, a congenital myasthenic syndrome. The congenital myasthenic syndromes are a genetically heterogeneous group of neuromuscular junction disorders. They most commonly present at birth, but some forms do not present until childhood or early adulthood. They are more common in males. The typical presentation is one of ophthalmoparesis and ptosis in infancy, with facial diparesis. Limb weakness and hypotonia are often present, and respiratory involvement is rare but can occur. As in autoimmune myasthenia gravis, edrophonium injection transiently improves symptoms in congenital myasthenia, and there is symptomatic benefit from acetylcholine esterase inhibitors. Because the congenital myasthenic syndromes are not immune-mediated, there is no response to immunosuppressive therapy or thymectomy. Electrophysiologic findings are similar to other neuromuscular disorders, showing a decremental response on repetitive nerve stimulation studies and increased jitter on single-fiber EMG. The most common form of congenital myasthenia, congenital acetylcholine receptor deficiency, is due to an autosomal recessive or sporadic mutation in the acetylcholine receptor gene.

Choline acetyltransferase deficiency is another congenital myasthenia syndrome that differs from other congenital myasthenic syndromes in that extraocular muscle involvement is not prominent, and the clinical presentation is more typically generalized hypotonia, apneas, and feeding difficulties. Slow-channel congenital myasthenic syndrome is a rare congenital myasthenia syndrome that is autosomal dominant in inheritance and results from abnormally prolonged opening of the acetylcholine channel at the postsynaptic membrane.

In newborns of mothers with autoimmune myasthenia gravis, transient myasthenic symptoms can occur due to transfer of maternal antibodies to the fetus in utero. Signs of neonatal myasthenia include hypotonia and poor feeding, with symptoms improving typically within the first 2 weeks of life. This patient’s symptoms were present at 6 months of age, and absence of evidence of a neuromuscular disorder in the patient’s mother makes this less likely.

Other disorders that can lead to ophthalmoparesis, ptosis, and generalized weakness in infancy include mitochondrial disorders, botulism, and congenital myopathies, but the presence of fluctuating weakness and response to edrophonium distinguish congenital myasthenia from these.

Bradley WG, Daroff RB, Fenichel GM, et al. Neurology in Clinical Practice, 5th ed. Philadelphia, PA: Elsevier; 2008.

Ropper AH, Samuels MA. Adams and Victor’s Principles of Neurology, 9th ed. New York: McGraw-Hill; 2009.

51. c, 52. d

This patient has Emery-Dreifuss muscular dystrophy. This condition is inherited in an X-linked fashion. The defect is in the gene encoding for the nuclear membrane protein emerin. There is an autosomal dominant form in which the gene affected, LMNA, encodes for the nuclear membrane protein laminin A/C.

Patients with Emery-Dreiffuss muscular dystrophy characteristically present with contractures, which can be seen at the elbows, ankles, and neck. Muscle weakness tends to affect the upper arms and shoulder girdle muscles first and later the pelvic girdle and distal leg muscles. There is no pseudohypertrophy of the calves, and intelligence quotient (IQ) is normal. Cardiac involvement is prominent, with serious conduction abnormalities, often requiring pacemaker placement.

Duchenne muscular dystrophy is discussed in questions 19 and 20. Becker muscular dystrophy is discussed in questions 30 and 31. Fascioscapulohumeral muscular dystrophy is discussed in question 9. Limb-girdle muscular dystrophy is discussed in questions 41 and 42.

Cardamone M, Darras BT, Ryan MM. Inherited myopathies and muscular dystrophies. Semin Neurol. 2008; 28:250–259.

Ropper AH, Samuels MA. Adams and Victor’s Principles of Neurology, 9th ed. New York: McGraw-Hill; 2009.

53. e

This patient has Ullrich’s congenital muscular dystrophy, which presents with neonatal weakness, contractures and distal hyperlaxity, as well as protrusion of the calcanei. It is associated with mutations of collagen type VI and is thought to be related to Bethlem myopathy.

Walker-Marburg syndrome is an autosomal recessive condition characterized by muscular dystrophy and brain and ocular abnormalities. Patients are hypotonic at birth with elevated creatine kinase levels. The ocular malformations include microphthalmia, colobomas, cataracts, glaucoma, corneal opacity, retinal dysplasia, and optic atrophy. There are multiple brain CNS malformations including hydrocephalus, aqueductal stenosis, cerebellar hypoplasia, and cortical abnormalities.

Muscle-eye-brain disease is an autosomal recessive condition in which there is also muscular dystrophy and brain and ocular abnormalities; however, the cortical changes are milder and the white matter changes are more focal. The eyes are also affected to a lesser degree than Walker-Marburg syndrome.

Fukuyama-type congenital muscular dystrophy and laminin-α-2 deficiency are discussed in question 8.

Bradley WG, Daroff RB, Fenichel GM, et al. Neurology in Clinical Practice, 5th ed. Philadelphia, PA: Elsevier; 2008.

Cardamone M, Darras BT, Ryan MM. Inherited myopathies and muscular dystrophies. Semin Neurol. 2008; 28:250–259.

Ropper AH, Samuels MA. Adams and Victor’s Principles of Neurology, 9th ed. New York: McGraw-Hill; 2009.

54. e

The parasympathetic nervous system can be divided into cranial and sacral portions (Figure 10.8). The cranial portion includes fibers that originate in brain stem nuclei and are relayed to their target sites via four cranial nerves: (1) oculomotor, (2) facial, (3) glossopharyngeal, and (4) vagus. In the parasympathetic nervous system, the ganglia are located close to the target site, with long preganglionic fibers and short postganglionic fibers. Cells located in S2 to S4 of the spinal cord give rise to the sacral portion of the parasympathetic nervous system and innervate the genitourinary system and distal colon. The neurotransmitter for all sympathetic and parasympathetic preganglionic fibers is acetylcholine that acts at nicotinic receptors in the ganglia. Parasympathetic postganglionic neurons predominantly release acetylcholine. The parasympathetic nervous system leads to vasodilation, bradycardia, production of bronchial secretions, gastric acid production, lacrimation, salivation, pupillary constriction, and erection.

FIGURE 10.8 The autonomic nervous system (Illustration by David Schumick, BS, CMI. Reprinted with permission of the Cleveland Clinic Center for Medical Art & Photography. © 2010. All Rights Reserved)

The intermediolateral cell column of the spinal cord, present from T1 to L2, contains the preganglionic neurons of the sympathetic nervous system. Small myelinated fibers known as white rami communicantes exit the intermediolateral cell column and synapse on sympathetic ganglia located in the paravertebral region. Postganglionic fibers known as gray rami communicantes are carried through spinal nerves T1 to T4 to supply thoracic regions and T5 to L2 to supply various subdiaphragmatic end organs including blood vessels, gastrointestinal organs, and genitourinary tract (Figure 10.8; sympathetic innervation to the head is carried by C8-T2, see questions 55 and 56 and Figure 10.9). The neurotransmitter released from postganglionic sympathetic fibers is norepinephrine, except in sweat glands, where acetylcholine is released and acts at muscarinic receptors. In the sympathetic nervous system, preganglionic fibers are short and postganglionic fibers are long (Figure 10.8).

Blumenfield H. Neuroanatomy through Clinical Cases. Sunderland, MA: Sinauer; 2002.

Ropper AH, Samuels MA. Adams and Victor’s Principles of Neurology, 9th ed. New York: McGraw-Hill; 2009.

55. e, 56. d

Horner’s syndrome results from a lesion in the sympathetic pathways (Figure 10.9; see also Figure 1.6). The sympathetic tracts originate in the hypothalamus and descend in the lateral brain stem to synapse with sympathetic neurons in the intermediolateral cell column in the spinal cord that is present from T1 to L2.

FIGURE 10.9 Cranial sympathetic innervation (Illustration by David Schumick, BS, CMI. Reprinted with permission of the Cleveland Clinic Center for Medical Art & Photography. © 2010. All Rights Reserved)

The face receives sympathetic innervation from intermediolateral cells at the C8 to T2 level. Preganglionic fibers synapse in the superior cervical ganglion. Postganglionic fibers travel along the common carotid artery. At the carotid bifurcation, fibers to the sphincter pupillae of the eye travel with the ICA, and are involved in pupillary dilation. Fibers to Muller’s smooth muscles in the upper eyelid are also carried along this route. Sympathetic fibers destined for the eccrine glands of the face travel along the ECA.

Horner’s syndrome, or oculosympathetic paresis, results from a lesion to the sympathetic pathways. Features include ptosis, due to loss of innervation of Muller’s smooth muscle, miosis, due to loss of innervation of the pupillary dilator muscle, apparent enophthalmos, due to lack of input to the lid retractors (the eye appears sunken, though it is not), and loss of ciliospinal reflex (absence of pupillary constriction when painful stimulation is applied to the cervical region ipsilateral to the lesion). In preganglionic lesions, such as those occurring from medullary infarction, during thoracotomy, or with large lung masses that impinge on the preganglionic fibers as they course toward the superior cervical ganglion, facial anhidrosis occurs as well. Lesions in the area of the ICA spare the fibers destined for the eccrine glands of the face, and anhidrosis does not occur.

The patient in question 55 has a Horner’s syndrome, and in the setting of chronic cough and the findings on examination, this suggests that this is due to an apical lung mass, or Pancoast tumor. The presence of anhidrosis makes an ICA dissection unlikely because fibers to facial sweat glands travel along the ECA (Figure 10.9). The patient in question 56 has an ICA dissection, and because the lesion is distal to branching of the sweat fibers to the face (which are, again, carried along the ECA), anhidrosis will not occur (see also Chapter 1 for further information on how lesions in different areas of the sympathetic pathways can be distinguished).

The ptosis and pupillary abnormalities seen in Horner’s syndrome can be distinguished from a third nerve palsy by the presence of intact extraocular movements of cranial nerve III-innervated muscles (medial rectus, inferior oblique, superior rectus, and inferior rectus). In addition, with loss of sympathetic innervation to the pupil, the anisocoria will be more prominent in dim light, as the contralateral (normal) eye will normally dilate, whereas the eye that has lost sympathetic innervation will remain miotic. The miotic eye will still normally constrict to light and accommodation (because parasympathetics are not impaired). Levator dehiscence is a common cause of ptosis in older adults, but pupillary involvement and the other features of Horner’s syndrome do not occur.

Blumenfield H. Neuroanatomy through Clinical Cases. Sunderland, MA: Sinauer; 2002.

Bradley WG, Daroff RB, Fenichel GM, et al. Neurology in Clinical Practice, 5th ed. Philadelphia, PA: Elsevier; 2008.

57. a

Dystrophic myotonia type 2 myotonic dystrophy is a proximal myopathy and is also known as PROMM or proximal myotonic myopathy.

The distal myopathies are a group of disorders that are slowly progressive and with onset most commonly in adult life, though earlier presentations may occur. There are weakness and atrophy of the muscles of the hands, forearms, and lower legs. Some are autosomal dominant and others are autosomal recessive, each caused by mutations in different genes.

The distal myopathies are shown in Tables 10.4 and 10.5, including the mode of inheritance, age of presentation, as well as gene product affected (if known).

TABLE 10.4 Autosomal recessive distal myopathies

TABLE 10.5 Autosomal dominant distal myopathies

Cardamone M, Darras BT, Ryan MM. Inherited myopathies and muscular dystrophies. Semin Neurol. 2008; 28:250–259.

Ropper AH, Samuels MA. Adams and Victor’s Principles of Neurology, 9th ed. New York: McGraw-Hill; 2009.

58. e, 59. e, 60. c

Penile erection is mediated by the parasympathetic nervous system and ejaculation by the sympathetic nervous system. In detrusor-sphincter dyssynergia, the detrusor muscle contracts against an unrelaxed urethral sphincter. The internal urethral sphincter is under sympathetic control.

Micturition (urination) has both voluntary and involuntary components. Voluntary contraction of the external urethral sphincter is mediated by sympathetic innervation derived from the intermediolateral cell column in the spinal cord at the level of L1 and L2 (Figure 10.8). The detrusor muscle is under involuntary control by the parasympathetic nervous system. The pontine micturition center regulates the detrusor reflex. Nerve roots S2 to S4 relay afferent sensory information from the genitourinary system, bladder, and anorectal area to the spinal cord. Somatic efferents to the skeletal muscles of the pelvic floor arise from the anterior horn cells at S2 to S4 and are carried by the pudendal nerves.

Normally, individuals are able to maintain voluntary control over micturition through the medial frontal micturition centers located in the paracentral lobules (the medial continuation of the precentral gyrus). At the initiation of voluntary micturition, the detrusor reflex, which is mediated by spinal cord circuits, is initiated by voluntary external urethral sphincter relaxation and involuntary relaxation of the internal urethral sphincter with detrusor contraction. If the external urethral sphincter is voluntarily contracted during micturition, the detrusor muscle involuntarily relaxes and micturition stops.

Urinary incontinence can result from lesions along various areas of the neuraxis. With lesions of the cauda equina (discussed in Chapter 11), a flaccid bladder results from loss of detrusor tone, sensation of bladder fullness is lost, and voluntary control over urination is lost. Overflow incontinence may occur; these patients often require intermittent self-catheterization chronically. With lesions affecting the paracentral lobule, such as hydrocephalus or tumors, voluntary control over the external urethral sphincter is lost. Lesions above the conus medullaris lead initially to a flaccid bladder leading to urinary retention with or without overflow incontinence. Some time after a spinal cord lesion, the bladder may become hyperreflexic. Urge incontinence results from overactive detrusor contraction; this is a frequent occurrence in patients with multiple sclerosis, though flaccid bladder with requirements for intermittent catheterization can also occur.

Penile erection is mediated by parasympathetic fibers derived from S3 and S4 and carried by the pudendal nerve. Ejaculation is a sympathetically mediated reflex arc: the afferent pathway is via the dorsal nerve of the penis and pudendal nerve to S3 and S4, and the efferent pathway includes the perineal branch of the pudendal nerve.

Blumenfield H. Neuroanatomy through Clinical Cases. Sunderland, MA: Sinauer; 2002.

Bradley WG, Daroff RB, Fenichel GM, et al. Neurology in Clinical Practice, 5th ed. Philadelphia, PA: Elsevier; 2008.

61. d

Hirschsprung’s disease is due to congenital absence of the myenteric plexus. The enteric nervous system consists of the myenteric (or Auerbach’s) plexus that is located between the outer and inner smooth muscle layers of the gastrointestinal tract and the submucous (or Meissner’s) plexus, which is located between the circular muscle layer and the mucosa. The myenteric plexus is predominantly involved in gut motility, whereas the submucous plexus is involved in secretory functions. Together, the myenteric and submucous plexus control the function of the gastrointestinal tract.

Hirschsprung’s disease is due to maldevelopment of the myenteric plexus. Most commonly, focal congenital absence of the myenteric plexus in the internal anal sphincter or rectosigmoid junction occurs, though in rare cases, the myenteric plexus may be absent throughout the gastrointestinal system. Segments of the colon that lack myenteric plexus cannot relax, leading to fecal retention and distention of proximal colonic segments. Some cases of Hirschsprung’s disease are due to mutations in the RET proto-oncogene.

Kandel ER, Schwartz JH, Jessel TM. Principles of Neural Science, 4th ed. New York: McGraw-Hill; 2000.

Ropper AH, Samuels MA. Adams and Victor’s Principles of Neurology, 9th ed. New York: McGraw-Hill; 2009.

62. c

Cardiac involvement is present in about 25% of the patients with myofibrillar myopathy.

Myofibrillar myopathy is a congenital muscular dystrophy, which may be autosomal dominant or recessive, and affects males and females equally. It is characterized by slowly progressive weakness of the muscles of the limbs and trunk, affecting both proximal and distal muscles, but more the lower than the upper extremities. Cardiac involvement with conduction abnormalities is present in about 25% of the cases, and peripheral neuropathy can also be seen. Hyporeflexia is frequent.

Causative mutations involve the proteins myotilin, desmin, and αβ-crystallin. Pathologically, there is focal dissolution of myofibrils and subsarcolemmal accumulation of dense granular and filamentous material, variation of fiber sizes, rimmed vacuoles, and central nucleation.

Cardamone M, Darras BT, Ryan MM. Inherited myopathies and muscular dystrophies. Semin Neurol. 2008; 28:250–259.

Ropper AH, Samuels MA. Adams and Victor’s Principles of Neurology, 9th ed. New York: McGraw-Hill; 2009.

63. c

This patient has myotonia congenita, more specifically Thomsen’s disease.

There are two types of myotonia congenita: Thomsen’s disease, which is autosomal dominant, and Becker’s disease, which is autosomal recessive. This group of disorders is caused by a channelopathy secondary to a mutation in the chloride channel gene CLCN1 on chromosome 7q. The main manifestation is myotonia, which is an impaired muscle relaxation as seen when the patients cannot relax their handgrip after grasping an object, and also manifested on percussion, leading to contraction and delayed relaxation. Myotonic potentials can be detected on EMG.

In myotonia congenita, as opposed to paramyotonia, there is a “warm-up” phenomenon, in which the myotonia improves after repetitive muscle activation.

Thomsen’s disease, as mentioned above, is autosomal dominant and begins in the first decade of life manifesting with painless myotonia, but not weakness, and is milder than the reces- sive form. Becker’s disease is the recessive form that presents later, usually in the second decade of life, is more severe than the dominant form, and may manifest with weakness after severe episodes of myotonia. Propofol and depolarizing neuromuscular blocking agents may aggravate the myotonia, and patients may have prolonged recovery. Mexiletine is the treatment of choice.

Paramyotonia congenita is discussed in question 17. Hyperkalemic and hypokalemic periodic paralysis are discussed in question 1.

Saperstein DS. Muscle channelopathies. Semin Neurol. 2008; 28:260–269.

64. b

This patient has critical illness myopathy (CIM) without polyneuropathy. CIM can occur independently or in combination with critical illness polyneuropathy (CIPN). The typical presentation is a flaccid weakness, which is diffuse and involves not only limb muscles but also the diaphragm, making these patients difficult to wean from mechanical ventilation. CIM occurs in severely ill patients, usually with systemic inflammatory response syndrome, and with risk factors including hyperglycemia, hypoalbuminemia, sepsis, exposure to steroids, neuromuscular-blocking agents, and certain antibiotics. Electrophysiologic studies show SNAP greater than 80% of the lower limit of normal and low-amplitude CMAPs, as well as myopathic findings on needle EMG. Pathologically, there is type II fiber atrophy and characteristic loss of thick myosin filaments.

CIPN presents in patients with similar risk factors as described above for CIM, and is characterized by the presence of axonal degeneration of motor and sensory fibers, as detected electrophysiologically by low SNAPs and CMAPs. Needle EMG may demonstrate neurogenic changes.

The management of these conditions begins with the treatment of the underlying cause and avoidance of risk factors. Adequate nutrition, physical therapy, and rehabilitation are necessary. Even though CIPN and CIM can occur independently, they are frequently seen together in critically ill patients. Because muscle regeneration is much faster than nerve regeneration, CIM improves earlier.

Bolton CF. Neuromuscular manifestations of critical illness. Muscle Nerve. 2005; 32:140–163.

65. c

This patient has Bethlem myopathy, which is an autosomal dominant condition associated with mutations affecting the gene encoding collagen type VI. These patients present with weakness and contractures of the elbow and ankles, as well as hyperextensible interphalangeal joints.

Walker-Marburg syndrome and muscle-eye-brain disease are discussed in question 53. Fukuyama-type congenital muscular dystrophy and laminin-α-2 deficiency are discussed in question 8.

Bradley WG, Daroff RB, Fenichel GM, et al. Neurology in Clinical Practice, 5th ed. Philadelphia, PA: Elsevier; 2008.

Cardamone M, Darras BT, Ryan MM. Inherited myopathies and muscular dystrophies. Semin Neurol. 2008; 28:250–259.

Ropper AH, Samuels MA. Adams and Victor’s Principles of Neurology, 9th ed. New York: McGraw-Hill; 2009.



If you find an error or have any questions, please email us at admin@doctorlib.org. Thank you!